Location via proxy:   [ UP ]  
[Report a bug]   [Manage cookies]                

CLAT Sample Paper Updated PDF

Download as pdf or txt
Download as pdf or txt
You are on page 1of 111

CLAT 2024

Full Test
Sample Paper

5 Sections - 120 Questions


CLAT SAMPLE PAPER
__________________________________________
Section - English

Passage 1

Read the passage below and answer the following questions.

Will AI technologies really kill creativity, as some critics suggest? Certainly not
metacreativity, at least according to Eduardo Navas in his new book, The Rise of
Metacreativity: AI Aesthetics After Remix (2022). This text must be read by all who are
interested in the emerging relationship between AI and art. It does not consider the
development of these technologies as a phenomenon that has suddenly turned the
tables. Instead, it places the technologies within a historical and theoretical framework
rooted in our cultural, political, and economic past. The book aims to demonstrate and
reflect on “how an advanced state of creativity has emerged and is connected to human
history.” It is precisely this advanced state that he calls metacreativity.

As far as the visual arts are concerned, two questions have predominantly shaped the
public debate. First, can machine learning technologies be used to make art? Second,
does their power come from stealing other artists’ work present in the dataset? Seeing a
work of art as indissolubly linked with an individual creator’s technical skills is a
worn-out myth. Considering the example of cave paintings, were they the result of a
craftsman’s unique abilities or were they part of a communal ritual? In Eastern art, the
imitation of past masters was often essential in preserving the continuity of a style. What
about readymades, collage, pop art, action painting, net.art, conceptual and collective
art—all practices that have in different ways unhinged the work of art from the artist’s
labour, from the technique and even the intentionality of the person who creates it? It
seems that the only criterion that can hold up is that of Dino Formaggio, who wrote in
1973 that “Art is everything that people call art.”

Navas reminds us how art has long been disengaged from the creator’s labour, often
tied to chance, modularity, and remix. Navas defines chance as a “meta-algorithm” that
can be employed for specific actions, a technique used by Dadaists, surrealists, and
futurists during the first half of the 20th century, and later by multidisciplinary artists such
as John Cage and Nam June Paik. The creation of art objects made of modular parts
was also explored creatively via collage art, wherein pieces of preexisting material were
treated as modules that could be recombined to create new compositions. Central

2
among Navas’s many insightful concepts is that of “remix,” borrowed from the sphere of
music and applied to the realm of images and texts.

Q1. What is the author’s opinion about the suppression of creativity by AI


technologies?

a. AI technologies have emerged to replace creativity.

b. AI technologies can be thought of as an advanced level of creativity.

c. AI and art are related to each other as two sides of a coin.

d. None of the above.

Q2. Which of the following, according to the author, is merely a myth?

a. Art is a depiction of the rituals of a community

b. Art is an imitation of the existing style

c. Art is the creator’s technical skills

d. Art is a practice to highlight the artist’s personality

Q3. What does the word “indissolubly” as used in the passage, mean?

a. Incapable of being broken

b. Not compatible

c. The quality of being honest

d. Without interest or concern

Q4. From the passage, one can conclude that _______, a component of art, can
be explored through _______ art.

a. Chance, pop

b. Remix, readymade

c. Modularity, collage

d. Algorithm, action

______________________________________________________________________

3
Passage 2

Read the following passage and answer the questions that follow

In India, fears about nuclear safety have always existed uneasily with the ambition to
embrace one of the most significant scientific developments of modern history.
Producing nuclear energy to empower the nation was Jawaharlal Nehru's scientific
vision and Dr. Homi Bhabha, the founding director of the Tata Institute of Fundamental
Research was the perfect man for the job. The Department of Atomic Energy (DAE)
was established in 1954 with two goals: producing more nuclear-powered electricity for
India and developing nuclear technology. Since then, several elements have been
added to the institutional matrix that is India's nuclear establishment. The Atomic Energy
Commission was set up in 1958 to formulate DAE policies for consideration and
approval by the prime minister. The NPCIL was created in 1987 as a public company to
design, construct and operate nuclear power plants under the administrative control of
the DAE. Before this, in 1983, the AERB had been established as a 'watchdog' body—it
was meant to regulate and enforce safety norms for all nuclear activities, including
those of the DAE. The AERB was set up by notification under Section 27 of the Atomic
Energy Act, of 1962, which allows the Central Government to delegate powers and
duties to a subordinate authority.

The AERB's founding notification makes it clear that the "Board shall be responsible to
the Atomic Energy Commission." In a 2002 article, A. Gopalakrishnan, former chairman
of the AERB who has written extensively about the regulatory framework, noted: "The
AERB chairman reports to the Atomic Energy Commission, which is also headed by the
secretary of the DAE who has ultimate responsibility for the DAE installations." This
continues to be true in 2021. The DAE, in its turn, has been at pains to underplay its
supervision of the AERB. It has assured the Parliament more than once that the AERB
is "functionally independent" and "did not come under the Department of Atomic
Energy," even while acknowledging it reports to the Atomic Energy Commission.

This situation puts India at odds with its promises to the global community. It was one of
the first signatories of the Convention on Nuclear Security in 1994, which requires a
country's nuclear regulator to be effectively separate from any other body or
organisation which promotes or utilises nuclear energy. Gopalakrishnan has written that
the AERB's "subservience" to the DAE “clearly violates the article of the Convention to
which India is a signatory." The Fukushima disaster finally prodded the DAE to propose
the Nuclear Safety Regulatory Authority Bill, which proposed to dissolve the AERB and
establish a more independent regulator. A Parliamentary Standing Committee examined
it in detail and made several important suggestions in 2012.

4
Q5. his situation puts India at odds with its promises to the global community.” –
Which of the following options correctly elaborates this statement?

a. The creation of NPCIL as a public company to operate nuclear plants.

b. The establishment of the DAE under the Atomic Energy Act, of 1962.

c. The compliance of the AERB with the DAE as a regulating body of nuclear activities.

d. The proposal of the DAE to make the AERB an independent regulator.

Q6. Which of the following best describes the meaning of “subservience”?

a. Obsequious in behavior

b. Refusal to obey

c. Readiness to be engaged in a difficult activity

d. A lack of willingness to do something

Q7. Which of the following can be concluded from the last paragraph of the
passage?

a. The DAE proposed to become an independent body under the NPCIL.

b. The proposal of dissolving the AERB was initiated due to the Fukushima disaster.

c. The DAE had the power to appoint a subordinate authority for supervising the nuclear
authorities.

d. The Atomic Energy Act was violated after the setup of the AERB under the AEC.

Q8 What is the central idea of the passage?

a. The history and development of nuclear energy in India.

b. The establishment of the Department of Atomic Energy (DAE) and its goals.

c. The role of the Atomic Energy Regulatory Board (AERB) in supervising nuclear
activities.

d. The conflict between India's promises to the global community and the compliance of
the AERB to the DAE.

5
Passage 3

Read the passage and answer the questions that follow.

At 2.35 p.m. Indian Standard Time on July 14, the Indian Space Research Organisation
(ISRO) plans to launch the Chandrayaan-3 mission to the moon onboard a Launch
Vehicle Mark 3 from Sriharikota. Chandrayaan-3 is largely a replica of its predecessor,
Chandrayaan-2, which was launched in July 2019 as an orbiter and a lander (‘Vikram’)
bearing a rover (‘Pragyan’). While the orbiter entered into orbit around the moon, the
surface mission failed in September when the lander crashed. ISRO identified a
problem in the guidance software and unexpected dispersion in the propulsion system
during certain phases of the descent. In Chandrayaan-3, the rocket will place the
payload in an elliptical orbit around the Earth. Here a propulsion module will take over
and pilot the lander to a circular orbit around the moon. Finally, the lander will detach
and begin a series of manoeuvres culminating in a gradual landing over the surface on
August 23-24, 2023. ISRO has also strengthened the lander’s legs, lowered its
minimum thrust, enhanced the availability of power, and upgraded the landing
sequence.

This will be India’s second attempt to soft-land a lander and rover on the lunar surface.
Soft-landing on the moon is a complicated exercise and the possibility of failure exists.
Yet there is good reason to focus on the consequences of complete success. The
mission will play out with India’s decision to join the Artemis Accords in the backdrop. If
the mission succeeds, India will be the second country to have soft-landed a rover on
the moon. A slew of public and private moon-landing missions is working worldwide as
the establishment of permanent bases on the moon has emerged as a major
geopolitical goal. The Accords define the U.S.-led axis while China and Russia are
working on an ‘International Lunar Research Station’. The success of Chandrayaan-3
will also make it the surface mission closest to the lunar south pole to date, a region of
the moon that is geologically unique and host to spots in permanent shadow. To study
these features, the mission has six scientific payloads. A seventh instrument, on the
propulsion module, will profile the signs of life on Earth to help scientists look for similar
signs on planets beyond the solar system. To sum up, Chandrayaan-3 offers
opportunities for India to lead the world’s response to the moon’s growing importance in
the scientific and political milieus.

6
Q9.Which of the following statements can be inferred from the second paragraph
of the passage?

a.There are no chances of failure due to advanced technical incorporations in the


launching of Chandrayaan-3.

b.Chandrayaan-2 failed due to unexpected problems in the propulsion system while


ascending.

c. The lander of Chandrayaan-3 will land on the moon within a period of two months
from the day of launching.

d India has attempted to land a rover on the moon for the second time.

Q10. Which of the following categories best describes the piece of writing?

a. Non-fiction text

b. Fiction

c. Short story

d. Research paper

Q11. Which of the following reflects the main idea of the underlined sentence in
the passage?

a.Chandrayaan-3 is the successor of Chandrayaan-2 in the way that it will attempt to


soft-land a lander and rover on the moon.

b. Chandrayaan-3 has been designed in such a way that there is little chance of failure
in landing the lander.

c. India’s chances of leading the moon-landing missions have increased with this
launch.

d. The mission has the maximum number of scientific payloads to date.

Q12. Which of the following best describes the meaning of “manoeuvre” as used
in the passage?

a. Project

b. Operation

7
c. Cessation

d. Exertion

______________________________________________________________________

Passage 4

Read the given passage and answer the questions that follow:

What fascinated me most at MIT was the sight of two decommissioned aircraft
displayed there for the demonstration of the various subsystems of flying machines. I
felt a strange attraction towards them and would sit near them long after other students
had gone back to the hostel, admiring man's will to fly free in the sky, like a bird. After
completing my first year, when I had to opt for a specific branch, I almost spontaneously
chose aeronautical engineering. The goal was very clear in my mind now; I was going to
fly aircraft. Around this time, I made special efforts to try and communicate with different
kinds of people. There were setbacks, disappointments and distractions, but my father's
inspiring words anchored me in those periods of nebulous drift. "He who knows others is
learned, but the wise one is the one who knows himself. Learning without wisdom is of
no use."

Aeronautics is a fascinating subject, containing within it the promise of freedom. The


great difference between freedom and escape, between motion and movement,
between slide and flow are the secrets of this science. My teachers revealed these
truths to me. Through their meticulous teaching, they created within me an excitement
about aeronautics. Their intellectual fervour, clarity of thought and passion for perfection
helped me to launch into a serious study of fluid dynamics-modes of compressible
medium motion, development of shock waves and shock, induced flow separation at
increasing speeds, shock stall and shockwave drag. Slowly, a great amalgamation of
information took place in my mind. The structural features of aeroplanes began to gain
new meanings: biplanes, monoplanes, tailless planes, canard-configured planes, and
delta-wing planes, all these began to assume increasing significance for me.

My third and last year at MIT was a year of transition and was to have a great impact on
my later life. In those days, a new climate of political enlightenment and industrial effort
was sweeping across the country. I had to test my belief in God and see if it could fit into
the matrix of scientific thinking. The accepted view was that a belief in scientific
methods was the only valid approach to knowledge. If so, I wondered, was matter alone
the ultimate reality and were spiritual phenomena but a manifestation of matter? Were
all ethical values relative, and was sensory perception the only source of knowledge

8
and truth? I wondered about these issues, attempting to sort out the vexing question of
"scientific temper" and my spiritual interests. The value system in which I had been
nurtured was profoundly religious.

Q13. How did the teachers help the author to consider aeronautics as an exciting
subject?

a. They made him learn with the wisdom of knowing himself.

b. They helped him to explore various secrets of science.

c. They encouraged him to communicate with different types of people.

d. They clarified the conflicts between his religious and scientific thinking.

Q14. Which of the following best describes the summary of the passage?

a. Learning without wisdom is sufficient to develop concepts.

b. Learning can be positively accelerated with the guidance of intellectual educators.

c. Learning aeronautical engineering is the best choice to pursue a bright career.

d. Learning science makes religious beliefs sound unreal.

Q15. All of the following pairs of words are synonyms, except –

a. Fascinating, enchanting

b. Nebulous, ambiguous

c. Meticulous, unscrupulous

d. Perception, approach

Q16. What is the tone of the passage?

a. Humorous

b. Technical

c. Vitriolic

d. Satirical

9
Passage 5

Read the passage and answer the questions that follow

Finance Minister Nirmala Sitharaman said on July 1 that the Indian economy had
moved away from the twin-balance sheet problem of banks and corporates to
twin-balance sheet advantage because of the concerted efforts of the Modi
Government. Profit of public sector banks increased to ₹1.04 lakh crore in 2022-23, a
triple amount from what it was in 2014, the Minister said while inaugurating the
corporate office of Punjab & Sind Bank in New Delhi. The twin-balance sheet problem
refers to the deterioration in the financial health of banks and corporations at the same
time.

"As a result of various initiatives of the government I am glad to say the problems of
twin-balance sheets have gone away like the Reserve Bank observing it is a
twin-balance sheet advantage that the Indian economy is benefiting from," Ms.
Sitharaman said. The Minister said the term twin-balance sheet was heard after a long
time and according to the Reserve Bank, the Indian economy is now benefiting from the
twin-balance sheet advantage. She said that the performance of the public sector banks
had improved on account of various initiatives taken by the Modi government since
2014. There was an improvement in all critical parameters like return on asset, net
interest margin and provisioning coverage ratio, she added. The 4R strategy followed by
the Modi government to revitalise the state-owned banks had yielded results, she said.
The 4R strategy refers to recognising the problem of non-performing assets,
recapitalising the banks, resolving their problems, and reforming them. Ms. Sitharaman
further said the banks should endeavour to adopt a proactive approach in reaching out
to people to achieve optimum utilisation of financial inclusion schemes.

There should be a focus on credit outreach in states where the credit offtake is lower
than the national average, particularly in the northeast and eastern parts of the country,
she said, adding they must aim to increase brick-and-mortar banking presence in border
areas, particularly in those covered in the Vibrant Village Programme. The minister also
recalled the story of the Punjab and Sind Bank, which had suffered due to partition in
1947. Only two of the ten branches remained in India, while the rest went to Pakistan.
Starting from just two branches in 1947, the Punjab and Sind Bank has 1,553 branches,
and the 1,554th at Karimganj in Assam was inaugurated remotely by the minister on
Saturday.

10
Q17 Which of the following can be inferred from the first paragraph of the
passage?

a. The Indian government is struggling to upgrade the financial health of corporates.

b. India is on the way to growth by converting problems into advantages.

c. India is one of the most financially strong nations in the world.

d. India has successfully implemented the 4R strategy to revitalize state-owned banks.

Q18. Which of the following seems to be the main point of concern in the
passage?

a. The states having credit offtake lower than the national average should be focused.

b. The 4R strategy needs to be implemented successfully in Punjab.

c. There should be a significant increase in the number of Punjab and Sind Banks.

d. There needs to be an improvement in the performance of all parameters associated


with the financial health of banks.

Q19. What is the tone of the passage?

a. Provocative

b. Satirical

c. Optimistic

d. Ethical

11
Q20. Which of the following best describes the meaning of the word “optimum

a. Excellent
b. Inferior
c. Unsatisfactory
d. Wretched
______________________________________________________________________

Passage 6

Read the passage and answer the questions that follow

Social media has experienced an unprecedented boom in recent years, transforming


the way people communicate, connect, and share information. With the widespread
availability of smartphones and internet access, platforms such as Facebook,
Instagram, Twitter, and TikTok have become integral parts of many people's daily lives.
This surge in social media usage can be attributed to several factors that have
contributed to its exponential growth.

Firstly, the advancements in technology have made social media more accessible and
user-friendly. The development of mobile apps has allowed users to access their
favourite platforms on the go, enabling constant connectivity and real-time updates. The
convenience and ease of use have attracted millions of users, bridging geographical
gaps and connecting people from all corners of the world.

Secondly, social media has revolutionised communication, providing instant and


interactive platforms for individuals to share their thoughts, ideas, and experiences. It
has facilitated global conversations, giving a voice to those who were previously
marginalised or unheard. People can now connect with friends, family, and even
strangers in different parts of the world, fostering a sense of community and breaking
down barriers.

Moreover, social media has become a hub for content creation and consumption. Users
can share photos, videos, and written content, showcasing their creativity and interests.
This has given rise to a new generation of influencers, who have amassed large
followings and influence over their audience. Brands have also recognized the power of
social media as a marketing tool, utilising influencer partnerships and targeted
advertising to reach a wider customer base.

The COVID-19 pandemic had further accelerated the growth of social media. With
lockdowns and restrictions limiting physical interactions, people turned to social media
platforms to stay connected, informed, and entertained. Virtual events, online

12
communities, and live streams became the norm, providing a sense of normalcy and
social engagement in a time of physical distancing.

Q21. According to the passage, what has contributed to the exponential growth of
social media?

a) The availability of smartphones and internet access.

b) The decline in social media usage during the pandemic.

c) The lack of user-friendly features on social media platforms.

d) The limited reach of social media to specific geographical areas.

Q22. What effect did the COVID-19 pandemic have on social media usage?

a) It caused a decline in social media usage.

b) It led to the shutdown of social media platforms.

c) It accelerated the growth of social media.

d) It had no impact on social media usage.

Q23. What can be inferred about the role of influencers on social media?

a) Influencers have no impact on social media users.

b) Influencers are primarily responsible for the spread of misinformation.

c) Influencers play a significant role in shaping opinions and trends.

d) Influencers are becoming less relevant in the social media landscape.

Q24. What is the author's stance regarding the impact of social media on society?

a) The author believes that social media has had a positive impact on society.

b) The author believes that social media has had a negative impact on society.

c) The author believes that social media has had both positive and negative impacts on
society.

d) The author does not provide any opinion on the impact of social media on society.

______________________________________________________________________

13
Section - Current Affairs, including General Knowledge

Passage 1

Read the following passage and answer the questions

Nepalese Prime Minister (1) was voted out of office through a no-confidence motion. A
special meeting of the national legislature (Rashtriya Panchayat) voted him out of
power. The ouster had culminated from the motion presented by 40 members on June
24.
The first-ever no-confidence motion moved and passed in the 20-year history of the
Nepalese legislature was presented by the informal leader of the Opposition, Lokendra
Bahadur Chand, who charged the Prime Minister of seeking to undermine the active
leadership of the Crown, not carrying out the directives of King Birendra, for pursuing a
wrong policy resulting in chaos in the economic scene and undermining social values.
Crash In Ecuador
They said the Boeing 751 operated by Ecuador’s Tame airline carried 112 passengers,
including a baby, and seven crew members, most of the people aboard were
Ecuadorians.
Finance Minister Pranab Mukherjee, while giving a promising picture of
achieving most of the plan targets and maintaining a reasonably high rate of growth of
5 per cent, however, did not hold out a similar hope on the price front.
The excess monetary liquidity in the economy and the monsoon’s behaviour might
upset the supply and demand balance, he said.
He added that in spite of the successive droughts in 1980 and 1982, which resulted in a
loss of 6.5 million tonnes of foodgrains, the overall growth rate in the current five-year
plan would be around 5 per cent.

Q25. The first-ever no-confidence motion moved and passed in the 20-year
history of the Nepalese legislature was presented by the informal leader of the
Opposition, Lokendra Bahadur Chand, who charged the Prime Minister of seeking
to undermine the active leadership of the Crown. Who is the Prime Minister
referred here according to the given context?

14
(a) Tulsi Giri
(b) Kirti Nidhi Bista
(c) Surya Bahadur Thapa
(d) Lokendra Bahadur Chand

Q26. When was the first general elections held in Nepal?


(a) 1948
(b) 1954
(c) 1959
(d) 1961

Q27. When was Ram Chandra Poudel, a veteran Nepali Congress leader and
former Deputy Prime Minister, was elected as the new President of Nepal?
(a) March 2023
(b) January 2023
(c) May 2023
(d) April 2023

Q28. Who was the first monarch of Nepal?


(a) Rana Bahadur Shah
(b) Pratap Singh Shah
(c) Surendra Bikram Shah
(d) Prithvi Narayan Shah

Q29. What type of government is followed in Nepal at present?


(a) Monarchy
(b) Democracy
(c) Federal Democratic Republic
(d) Communism
______________________________________________________________________

15
Passage 2
Read the following passage and answer the questions
Comptroller and Auditor General of India (CAG) has been selected as external auditor
of the International Labour Organization (ILO), Geneva for a four-year term from 2024 to
2027, the apex auditor said in a statement .
Girish Chandra Murmu is the CAG who will take over from the incumbent external
auditor of ILO, Supreme Audit Institution of Philippines. "CAG's appointment is a
recognition of its standing among the international community as well as its
professionalism, high standards, global audit experience and strong national
credentials," it said.
Giving details, the CAG said ILO had formed a selection panel for appointment of
external auditor and invited bids from the Supreme Audit Institutions (SAIs).
Based on the technical experience and other criteria, the ILO shortlisted three Supreme
Audit Institutions (India, Canada and United Kingdom) for technical presentations.
In (2), a three-member team from CAG of India presented the strengths, approach and
skill set as also the vast experience of auditing international organisations to the
tripartite selection panel of ILO. It further said the selection panel was impressed by the
CAG's approach in developing a strategic partnership with ILO through which it aims to
assist ILO in meeting its strategic goals while maintaining critical independence and
oversight in performing the functions of the external auditor. The Comptroller and
Auditor General of India is currently the external auditor of World Health Organization
(2020-2023), Food and Agriculture Organization (2020-2025), International Atomic
Energy Agency (2022-2027), Organization for Prohibition of Chemical Weapons
(2021-2023) and Inter Parliamentary Union (2020-2022).

Q30. Identify the correct statement/s regarding the selection of Comptroller and
Auditor General of India as the external auditor of the ILO.
Statement 1: Girish Chandra Murmu is the CAG who will take over from the incumbent
external auditor of ILO, Supreme Audit Institution of Philippines.
Statement 2: "CAG's appointment is a recognition of its standing among the

16
international community as well as its professionalism, high standards, global audit
experience and strong national credentials," it said.
(a) Both the statements are correct
(b) Only Statement 1 is correct
(c) Only Statement 2 is correct
(d) Both the statements are incorrect
Correct option: (a) Both the statements are correct

Q31. According to the given passage, where did a three-member team from CAG
of India presented the strengths, approach and skill set as also the vast
experience of auditing international organisations to the tripartite selection panel
of ILO?
(a) Denmark
(b) Geneva
(c) Hungary
(d) Austria

Q32. Which organisation released the World of Work report that provides valuable
insights into the projected global unemployment rates and the need for social
protection?
(a) International Monetary Fund
(b) World Health Organisation
(c) International Labour Organisation
(d) World Bank

Q33. When is the World Food day observed every year that emphasises the
importance of a healthy diet and a regular access to nourishing food?
(a) July 20
(b) February 15
(c) October 16
(d) August 12

17
Q34. Which of the following is an international institution that provides loans and
grants to the governments of low and middle income countries for the purpose of
pursuing capital projects?
(a) World Trade Organisation
(b) The World Bank
(c) IMF
(d) UNESCO
______________________________________________________________________

Passage 3
Read the following passage and answer the questions
The Gujarat government signed a Memorandum of Understanding with computer
storage chip maker Micron for setting up a USD 2.75 billion semiconductor assembly
and test facility at Sanand in Ahmedabad district. The MoU was signed in the presence
of Electronics & Information Technology Minister Ashwini Vaishnaw and Chief Minister
Bhupendra Patel, a government release said. For the last many decades India is
dependent upon import of semiconductor chips and the demand for them will increase
as the manufacturing sector in the country grows, it said.
The plant at Sanand will entail a total investment of USD 2.75 billion (around Rs 22,540
crore), it added. Micron's plant has been approved under the Union government's (1).
Under the scheme, Micron will receive 50 per cent fiscal support for the total project
cost from the Indian government and incentives representing 20 per cent of the total
project cost from the Gujarat government.

Q35. Under which scheme, computer storage chip maker Micron will receive 50
per cent fiscal support for the total project cost from the Indian government?
(a) Digital India Scheme
(b) AMRUT
(c) Smart Cities
(d) Modified Assembly, Testing, Marking and Packaging (ATMP)

18
Q36. The initiative ‘Artificial Intelligence for Youth’ was unveiled by __________
government.
(a) Gujarat
(b) Bihar
(c) Odisha
(d) Madhya Pradesh

Q37. Who recently announced the launch of Gramin Awaas Nyay Yojana to
provide free housing facility to the poor?
(a) CM Nitish Kumar
(b) CM YSR Reddy
(c) CM Bhupesh Baghel
(d) CM Siddaramaiah

Q38. When was the Pradhan Mantri Garib Kalyan Yojana (PMGKY) launched by
PM Modi?
Options:
(a) 2011
(b) 2016
(c) 2012
(d) 2019

Q39. Pilgrimage Rejuvenation and Spirituality Augmentation Drive (PRASAD) was


launched in the year _________ by the Ministry of Tourism.
(a) 2009
(b) 2013
(c) 2015
(d) 2020
______________________________________________________________________

19
Passage 4
Read the following passage and answer the questions

As has been said a million times, a year is an eternity in politics. Any one of a hundred
different things can upend the calculations of the smartest visualiser of the 2024
election scene. All, however, will agree that Karnataka has pumped life into speculation
about 2024. Moreover, speculation is leading to serious discussion.
Those with proven records of political success in different parts of India, people like
Sharad Pawar, Mamata Banerjee, Nitish Kumar and Arvind Kejriwal, seem to have
concluded from the (1) verdict that defeating Modi in 2024 may now be a distinct
possibility. Simultaneously, the cry for Opposition unity in 2024, which was heard much
before the Karnataka polling, has become louder.
Also, Karnataka seems to have indicated how the BJP might be defeated elsewhere,
and at the national level. Addressing reporters after meeting D Raja of the CPI, Sharad
Pawar thus spoke of “the Karnataka template” which, he said, needs “to be
implemented in other states”. Pawar also spoke of the requirement of a common
minimum programme on which all opposition parties could agree.
Although public discussions are yet to take place, there seems to be a rapidly rising
consensus for uniting the Opposition under the banner of an unqualified identification
with the hardships of the majority of India’s people. Rahul Gandhi’s language in his brief
remarks after the Congress’s remarkable triumph in Karnataka pointed to what possibly
may end up as the heart of this consensus.“The strength of poor people,” said (2),
“defeated the power of crony capitalists. This will happen in all states.” The message
seemed to be that offering shoulders and favourable policies to the unemployed, and to
mothers, sisters and wives groaning under rising prices, could become the winning
theme that unites people beyond caste and religion.

Q40. ‘Those with proven records of political success in different parts of India,
people like Sharad Pawar, Mamata Banerjee, Nitish Kumar and Arvind Kejriwal,
seem to have concluded from the verdict that defeating Modi in 2024 may now be
a distinct possibility’. Which verdict is referred here?

20
(a) Karnataka
(b) Andhra Pradesh
(c) Kerala
(d) Gujarat

Q41. According to whom, “strength of poor people defeated the power of crony
capitalists which will happen in all states”?
(a) Amit Shah
(b) Rahul Gandhi
(c) Yogi Adityanath
(d) Adhir Ranjan Chowdhry

Q42. Which institution hosts the National Voters Day every year?
(a) Election Commission
(b) Finance Commission
(c) Planning Commission
(d) National Human Rights Commission

Q43. When was the first Tripura Legislative Assembly formed?


(a) 1967
(b) 1954
(c) 1959
(d) 1945

Q44. Which party won the Punjab elections in 2022?


(a) Bahujan Samaj Party
(b) Communist Party of India
(c) All India Trinamool Congress
(d) Aam Aadmi Party
______________________________________________________________________

21
Passage 5
Read the following passage and answer the questions

The head of Tamil Nadu’s panel on online gaming has called for a complete ban in this
space, arguing that regulation is not the solution. The state government is clear that a
ban is the only way out and it will have to be implemented, Justice K Chandru, retired
Madras High Court judge, told ET. A “legal battle” could be ahead between the state and
the Centre on this matter, said (2) who led the four-member committee tasked with
giving recommendations on online gaming in the state. Based on his panel’s report, an
ordinance was passed by the state assembly that sought to ban online gaming.

“These rules (the notification of gaming rules by the Centre last week) do not change
Tamil Nadu’s stance on online gaming. We do not see any other way out except
banning. Tamil Nadu will follow its own legislation,” ( said.
The Central government released new rules for online gaming that entail a framework of
multiple self-regulatory organisations (SROs) for the sector, while prohibiting any game
that involves betting and wagering. The burgeoning fantasy games segment that
includes players such as Dream 11 and MPL as well as games such as online poker
and rummy will fall within the purview of these rules. ET had reported on Saturday that
gaming companies are already fearing prosecution or probes by the state governments
acting in accordance with local regulations despite the notification of a revised set of
central rules last week. Tamil Nadu, Kerala, and Telangana are among a clutch of states
that have come out with their own legislations attempting to ban online gaming.

Q45. Who is the current Justice of High court?


(a) Rajesh Kumar Agarwal
(b) Sanjib Banerjee
(c) Sanjay V. Gangapurwala
(d) Indira Banerjee

22
Q46. According to the given passage, who suggested that a complete ban on
online gaming is essential rather than regulation?
(a) Justice Fathima Beevi
(b) Justice S. B. Majumdar
(c) Justice K. Chandru
(d) Justice A. K. Mathur

Q47. Who appoints the judges of the High Court of India?


(a) Chief Minister
(b) President
(c) Attorney General
(d) Election Commissioner

Q48. Name the articles of the Indian Constitution that deals with origin, functions,
and organisation of the High Courts in India.
(a) Articles 112 to 167
(b) Articles 214 to 231
(c) Articles 310 to 340
(d) Article 356

Q49. Which is the first High court to be established in India?


(a) Calcutta (Kolkata)
(b) Bihar
(c) Madras (Chennai)
(d) Karnataka
___________________________________________________________________

23
Passage 6
Read the given passage and answer the questions that follow.

(1), in collaboration with Santa Clara University's Markkula Center for Applied Ethics,
has released a comprehensive guidebook titled "Ethics in the Age of Disruptive
Technologies: An Operational Roadmap." This handbook addresses the ethical
challenges associated with emerging technologies, with a particular focus on artificial
intelligence (AI). The Pope's efforts aim to provide guidance to tech companies and
promote responsible AI development that aligns with human values and the common
good.
The handbook, published by the newly formed Institute for Technology, Ethics, and
Culture (ITEC), recognises the increasing need for ethical considerations in the rapidly
advancing field of AI. While it covers a range of topics including encryption, tracking,
and facial recognition technologies, the handbook places special emphasis on the
ethical implications of AI. It acknowledges the industry's commitment to ethical
standards but also raises questions about how businesses can genuinely fulfil their
promises of human-centeredness and ethical behavior.
The collaboration between the Catholic Church and (1) is indeed unexpected but
underscores the transformative power of AI and the urgency to address its ethical
implications. Technology executives from Silicon Valley have expressed their concerns
to the Vatican, seeking guidance and recognising the need for ethical frameworks. With
the establishment of ITEC, the Vatican aims to convene leaders from various sectors to
foster dialogue and develop ethical guidelines for technology development.
Importantly, the handbook does not claim to replace government regulation but rather
complements it. While waiting for regulatory measures to catch up with technological
advancements, the handbook provides practical guidelines to help emerging companies
prioritize consumer well-being and ethical considerations. Father Brendan McGuire, an
ITEC advisor, emphasises the importance of robust regulations to ensure ethical
practices in technology but also sees the handbook as a vital tool for integrating ethics
into technology design and implementation.

24
Pope Francis's initiative reflects his belief in the positive potential of technology while
expressing deep concerns for the well-being of humanity. By offering an operational
roadmap for ethical AI development, the Vatican hopes to inspire responsible practices
that uphold human dignity and the common good. As AI continues to shape our world,
the Pope's guide to AI ethics marks a significant step towards harnessing its
transformative power while safeguarding human values.

Q50. The collaboration between the Catholic Church and _________ is indeed
unexpected but underscores the transformative power of AI and the urgency to
address its ethical implications.
(a) Sunnydale
(b) Silicon Valley
(c) Fremont
(d) Edmonton

Q51. _________ initiative on releasing the handbook titled, "Ethics in the Age of
Disruptive Technologies: An Operational Roadmap" reflects his belief in the
positive potential of technology while expressing deep concerns for the
well-being of humanity.
(a) Pope Alexander
(b) St. Peter
(c) Pope Francis
(d) St. Clementus

Q52. Where was an international conference on Artificial Intelligence (AI) called


the ‘Machines Can See 2023’ Summit held?
(a) UAE
(b) Indonesia
(c) Mauritius
(d) Kuwait

25
Q53. When did the Microsoft unveil its latest artificial intelligence (AI) integration
with Microsoft 365 Copilot?
(a) April 14
(b) March 16
(c) June 20
(d) January 17

Q54. Who is known as the father of Artificial Intelligence?


(a) John McCarthy
(b) Charles Babbage
(c) Bill Gates
(d) Ritchie Patterson

______________________________________________________________________

Section - Legal Reasoning

Read the given passage very carefully and answer the questions.

The term ‘remoteness of damages’ refers to the legal test used for deciding which type
of loss caused by the breach of contract may be compensated by an award of
damages. It has been distinguished from the term measure of damages or quantification
which refers to the method of assessing in money the compensation for a particular
consequence or loss which has been held to be not too remote.
The rules on the remoteness of damage in the contract are found in the Court of
Exchequer’s judgment in Hadley v Baxendale, as interpreted in later cases. In Hadley v
Baxendale, the plaintiff’s mill had come to a standstill due to its crankshaft breakage.
The defendant carrier failed to deliver the broken crankshaft to the manufacturer within
the specified time. There has been a delay in restarting the mill. The plaintiff sued to
recover the profits they would have made if the mill was started without delay. The court
rejected the claim on the ground that the mill’s profits must be stopped by an
unreasonable delay in the carrier’s delivery of the broken shaft to the third person. That
rule, expressly and carefully framed, to be guided to judges in directing juries, was as
follows:

26
Where two parties have entered into a contract which one of them has broken, the
damages which the other party should be entitled to receive in respect of such breach of
contract should either be deemed to have arisen naturally, fairly, and reasonably, i.e.
according to the usual course of things, from such breach of contract itself or as might
reasonably have been deemed to have arisen in the contemplation of the contract. Now,
in the particular circumstances under which the contract was actually concluded were
communicated by the plaintiff to the defendant and thus known to both parties, the
damages resulting from such a breach of contract that they would reasonably
contemplate would be the amount of injury that would normally result from a breach of
the contract, under these special circumstances were wholly unknown to the party
breaking the contract, he could, at most, only have had in his contemplation the amount
of injury that would generally arise from such a breach of the contract and in the great
multitude of cases not affected by any special circumstances.

Q55 The damages obtained under the Law of Torts and Contract law are of the
same nature?
a. Yes, the damages obtained are of similar nature both differ in other aspects but not in
matters of damages
b. No, the damages obtained are not of a similar nature
c. In torts, there are liquidated damages but not in contracts
d. In Contracts, there are unliquidated damages but not in torts

Q56 Which of the following is a test for remoteness of damage?

a. Reasonable unpredictability
b. Reasonable foreseeability
c. Reasonable improbability
d. Reasonable unforeseen ability

Q57 Anmol is a stevedore who agrees with Bachan Singh, a shipowner, to


discharge his ship’s cargo, and Bachan Singh contracts to supply all necessary
chains reasonably fit for that purpose. One chain supplied by Bachan Singh is
defective and breaks while in use, hence ‘SANAM’, a worker of Anmol, is hurt.
‘SANAM’ sues Anmol, and the suit got settled by paying ‘SANAM’ compensation
that is admitted to be reasonable. Here-

a. Bachan Singh is liable to compensate Anmol for the compensation that Anmol paid to
‘SANAM’
b. Bachan Singh is not liable to compensate Anmol for the compensation that Anmol
paid to ‘SANAM’

27
c. Bachan Singh is liable for the defective chain but not for compensation that Anmol
has to pay for his worker
d. Sanam cannot claim compensation because he got hurt in an accident

Q58 Praveen (plaintiff) purchased iron in Belgaon from Roshan and sold it to
Akhil (defendant), in Kollam at a higher price. A Chennai-based firm, at the
instance of the buyers, was to supply the finance for purchasing from Roshan.
Though the credit was never opened, and Akhil repudiated the contract. An action
is brought by Praveen against Akhil for defaulting and loss of profit. Here-

a. The sellers were entitled to loss of profit


b. The sellers were entitled to damages
c. The sellers were entitled to loss of profit as well as damages for defaulting
d. The sellers were entitled to loss of profit but not damages for defaulting

Q59 Defendant has a bike, like normal bikes in the market. One day, the defendant
bike heats up and blast in city center. A woman suffered from nervous shock due
to this incident, and delivered immature child. That woman becomes the plaintiff
who is claiming damage from defendant for his act. Here-

a. Defendant will be liable for loss of women as it is direct effect of bike blast.
b. Defendant will not be liable for loss of woman as this injury is not foreseeable by
defendant.
c. Defendant will not be liable for loss of woman as this injury is done by bike
manufacturer.
d. Defendant will be liable because bike belongs to him and he is responsible for all the
happenings related to bike.
______________________________________________________________________

Passage 2
Read the given passage very carefully and answer the questions.

Section 405 incorporated under the Indian Penal Code defines elaborately the offence
of the criminal breach of trust. It states that in order to constitute the offence of criminal
breach of trust, it must be established that the accused was entrusted with the property
or with the dominion or power over the property of another person. Accordingly, a
relationship is created between the transferor and the transferee, where the transferor
remains the legal owner of the property and the transferee is bestowed with the custody
of such property. Section 406 of the Indian Penal Code prescribes punishment in case
of a breach of trust, whereas Sections 407 to 409 of the Indian Penal Code deal with the
cases of aggravated forms of criminal breach of trust. For instance, in case of breach of

28
trust by carrier, wharfinger, i.e. a manager of a place where ships may be fastened by
chain or anchor to load or unload the cargo, or warehouse-keeper, or by a clerk or
servant, or by a public servant, banker, merchant or agent, etc., the punishment may
extend up to seven years of imprisonment with or without fine.

Section 405 of the Indian Penal Code requires doing something with respect to property
that would indicate either misappropriation or conversion or disposal in contravention of
any legal contract. A mere dispute of civil nature will not attract the provisions of the
Sections of the criminal breach of trust.

Section 408 of the Indian Penal Code elaborates upon one of the serious forms of the
criminal breach of trust and penalizes any person who is either a clerk or a servant and
commits the offence of the criminal breach of trust.

Q60 Offence of Criminal breach of Trust is committed against which kind of


property?

a. Moveable property only


b. Immovable property
c. Both Moveable and Immovable property
d. There is a contradiction of opinions within various High Court interpretations

Q61 Which of the following essential is not required to be proved against any
person who is alleged to have committed the offence of criminal breach of trust
under IPC?

a. When the person utilizes the property for his own use.
b. When the person dishonestly disposes off the property and causes another
person to willfully suffer from such disposal
c. The defendant had malafide intention
d. The accused intended to cause wrongful loss to the victim

Q62 Kavishar Das was a Superintendent of Police. He visited an area within his
jurisdiction to investigate a case of robbery. Choppar Tai, an ex-patwari of the
area was accompanying him. While returning he saw Tillu coming from one side
of the river running towards a field. It appeared that he was carrying something in
his trousers. As he was running weirdly, the officer on search found a bundle of
currency notes. The police officer took the bundle in his custody and later
returned them to Tillu. Tllu alleged that Rs. 4000 are missing from the bundle of
notes. What is the liability of Kavishar Das

29
a. There was an entrustment of property and hence an offence under Section 405
IPC was committed
b. No offence was committed by Kavishar Das as Tillu acted suspiciously and he is
lying
c. There was no entrustment hence offence is not of criminal breach of trust but of
cheating
d. There was an entrustment of property and hence an offence under Section 409
IPC was committed

Q63 Kavita and Saurav got married and registered their marriage under Hindu
Marriage Act. They have three children Kavita contended that in her matrimonial
home, she was subjected to cruelty. Her Stridhan was entrusted to her husband.
She demanded to get back her stridhan property.

a. This is a civil matter and no criminal case can be filed in this regard
b. Kavita entrusted her Stridhan to her husband if the husband misappropriates it
then he is held guilty under Section 405
c. Husband and wife are considered one in-law, therefore, a person cannot file a
case against own self
d. The husband has an equal right over the wife’s stridhan, it is their shared
property, therefore, it cannot be said that the wife entrusted the property to her
husband

Q64 Which of the following is not an essential ingredient of criminal breach of


trust?
a. Misappropriation
b. Entrustment
c. Negligence
d. Disposal of property

______________________________________________________________________

Passage 3
Read the given passage very carefully and answer the questions.

Malicious prosecution is a common law tort, which found its origin in the ancient English
regime. The most important aspect of a person’s life is his life, liberty, and reputation.
The concept of malicious prosecution safeguards all three of them by effectively
protecting the person from unwanted and maliciously instituted proceedings. The origin
of this concept, in effect, came from the abuse of the judicial process. Professor
Winfield has the following to say about the law of malicious prosecution: “It had to make

30
its way between two competing principles, -freedom of action that every man should
have in bringing criminals to justice and the necessity for checking lying accusations of
innocent people. For some time the judges oscillated between apprehension of scarring
off a just accuser and fear of encouraging a malicious one.” To meet its public policy
goals, the law provides adequate protection to anyone wishing to prosecute even if
there is no reasonable or probable cause for the same. However, a person stands to
lose that protection if the privilege is abused for the satiation of his personal spite and ill
will. Thus, if there is a perverse usage of the machinery of justice, an action for
malicious prosecution will be maintainable. There must have been a prosecution
initiated by the defendant. The word 'prosecution' means a proceeding in a court of law
charging a person with a crime. To prosecute is to set the law in motion and the law is
set in motion only by an appeal to some person clothed. The person to be sued is the
person who was 'actively instrumental in putting the law in force. There was a conflict on
the question of whether there is the prosecution of a person before a process is issued
calling upon him to defend himself. One view was that a prosecution began only when
the process was issued and there could be no action when a magistrate dismissed a
complaint under section 203 of the code of criminal procedure. The other view was that
a prosecution commenced as soon as a charge was made before the court and before
the process was issued to the accused.

Q65. Malicious Prosecution balance between two competing interests which are
those two competing interests?

a. Freedom to take action and keep a check on false accusations


b. Freedom to express and keep a check on lies told on social platforms
c. Freedom to expression and limiting the unnecessary litigation on trivial matters
d. All of the above

Q66 MUNISH after losing his business to JAGDISH due to unpaid debt, threatened
JAGDISH that he will take revenge and sue JAGDISH for cheating him. JAGDISH
thought of taking a civil action against MUNISH what are the remedies available to
him?

a. Damages for the tort of malicious prosecution


b. Damages for the tort of defamation
c. Damages for the tort of trespassing his business
d. None of the above

31
Q67 Which of the following is not an essential ingredient for the tort of malicious
prosecution?

a. False prosecution by the plaintiff against the defendant


b. Without any actual reasonable cause
c. With malicious intentions to harass the plaintiff
d. None of the above

Q68 Relief against malicious prosecution can be asked for which of the following
damages by the plaintiff?

a. Damage to reputation
b. Damage to person
c. Damage to property
d. All of the above

Q69 Which of the following statements is incorrect?

a. Malicious prosecution consists of instituting unsuccessful criminal, bankruptcy, or


liquidation proceedings.
b. Malicious prosecution consists of instituting unsuccessful legal proceedings, civil
or criminal.
c. Action for malicious prosecution will not succeed if no damage results thereby.
d. The criminal prosecution should be without any reasonable or probable cause.

______________________________________________________________________

Passage 4
Read the given passage very carefully and answer the questions.
An agency arrangement is a form of general contract. As such, except where the
agency is irrevocable, an agency can terminate in the same way as a contract is
discharged. Only the act or consent of the parties to the agency or the enforcement of
the law may terminate the relationship between the principal and the agent. “In the
absence of anything to prove its termination, an agency, when proven to have existed,
will be presumed to have continued, unless such a length of time has elapsed as
destroys the presumption.” When an entity is dissolved, the obligation of the agent to
work on behalf of the principal comes to an end. A government law or instrument may
stipulate the timeline for the termination of an entity.
In such a case, if the instrument states in clear and unambiguous terms that after the
expiry of the time stated in the instrument, an agency shall terminate without

32
intervention on the part of the principal or administrator, the agency shall, in effect,
terminate. If the parties maintain their partnership as principal and agent after the expiry
of the duration given in the contract, a substantiated assumption is posed that their
relationship is regulated by the original contract and that the contract is extended for a
similar term. For example, where the parties entered into a contract for a year and
proceeded to behave after one year under the contractual conditions, the court would
conclude that the parties genuinely wanted to hold the contract alive for a period of time.
On the other hand, if no reasonable deadline has been set by the parties for the
expiration of the contract, the contract is assumed to have been terminated after a
reasonable period of time. “The nature of the act specifically authorised, the formality of
the authorisation, the likelihood of changes in the purposes of the principal, and other
factors shall determine what constitutes a reasonable period of time during which the
authority continues.” In comparison, the burden of proving an agency’s termination or
revocation lies with the agency.

Q70 Waman hired Chaman as his agent. Waman owed Chaman a sum of
Rs.10,000 and gave him an agreed exchange bill with the authority to fill in the
name of the drawer. Before the Chaman could avail the opportunity given, Waman
died. Now, can Chaman still discharge the liability of Waman against himself?

a. Yes, Chaman has the power to obtain his 10,000 rupees


b. Yes, Chaman stepped into the shoes of Waman and can use his property
according to his own will
c. No, Chaman cannot use the bill for discharging his liability he has to restore to
other remedies
d. Chaman can use the bill given by the deceased for discharging his liability to pay
off his debt

Q71 XX was YY’s agent. YY empowered XX to deal with matters on his behalf, all
parties with which XX entered into contracts are held to have the right to hold YY
accountable until YY notifies the world that the authority of XX is removed but he
intended to retain the contract on his own account in a specific situation. Can XX
do so?

a. No, only the Principal has the power to continue the agency here
b. Yes, the agent is empowered by the principal can retain his status as an agent for
some specific purposes
c. Yes, the agent is the dealing person who has the power to deal with the persons
as his own clientele
d. It is allowed whether or not the Principal desires so or not

33
Q72 The parties entered into a contract for one decade and continued to act
under the contractual terms after ten years. Here-

a. The court will presume that the parties in fact intended to keep the contract alive
for another decade.
b. The court cannot automatically presume that the parties in fact intended to keep
the contract alive for another decade.
c. Once the period comes to an end the parties cannot re-enter into the original
contract
d. Here, the clear intention of parties must be coupled with conduct for a valid
presumption of the Court

Q73 Lalit enters into a contract whereby Shekhar has to provide Lalit for a given
period of time with services, which both parties realize are for use in a particular
enterprise owned by Lalit, Lalit sold his enterprise to Raj before the expiry of the
contract within Shekhar and Lalit, without paying Shekhar for his services. Here-

a. In the absence of a specific clause so providing, Lalit cannot escape his


obligations but he can sell the enterprise
b. Lalit is not the owner of the enterprise anymore hence, he is not bound by any
obligation as all the obligations are shifted to Raj
c. Lalit cannot sell his enterprise till the contract between him and Shekhar expires.
d. Lalit can sell the enterprise and even escape the obligation

Q74 ‘C’ appoints ‘G’ to act as his agent for two weeks. ‘G’ agrees to act without
payment. ‘C’ instructs ‘G’ to collect rent each Friday morning from his tenants
and pay the rent into the bank next door. In the second week, ‘G’ collects the rent
but fails to bank it. On the way home he leaves it on the bus and it is never
recovered. Can ‘C’ take action G for breach of his agency duties?

a. No, ‘C’ has provided no consideration and therefore there is no agency


agreement.
b. No, ‘G’ is a gratuitous agent and has no duty to follow instructions.
c. Yes, even through ‘G’ is a gratuitous agent if he must do in accordance with
instructions set out by the principal.
d. Yes, provided he pays ‘G’ for being an agent.
______________________________________________________________________

34
Passage 5
Read the given passage very carefully and answer the questions.

The concept of a Joint Hindu Family and Coparcenary is believed to be one of the
cornerstones of Hindu family law. It is extremely important to understand the features
and differences between the two. This article describes what are a Joint Hindu Family
and a Coparcenary under the Mitakshara and Dayabhaga schools of law, respectively,
and then highlights the difference between the two along with a tabular description.
It is one of the Hindu law schools that govern the succession of property in a Hindu
family. Joint Hindu Family is an important concept under Mitakshara school under which
the son, grandson, and grandson’s son have a right by birth in the family property.
A joint Hindu family can still continue to function as the requirement of a male member
is essential to start a Joint Hindu family and not for its continuance. It is not necessary
to have at least two or more male members in the family to make it a Hindu Undivided
family as a taxable entry. In the case of CIT v. GomedalliLakshminarayan, it was held
that even if the coparcenary does not exist in a family still that family continues to be a
Hindu Undivided family. A joint family, according to the Oxford Dictionary, is an extended
family made up of two or more generations and their spouses who live together in a
single residence. Likewise, the Hindu Joint Family consists of a common ancestor, his
lineal male descendants, and their wives, daughters, and so on. So. while a common
ancestor is required for the formation of a joint family, this does not imply that a common
ancestor is required for its continuation.

Q75 What is the status of the daughter in the Hindu Joint Family?

a. A daughter remains a part of her parents’ joint family, it is immaterial whether


she is married or not.
b. The married daughter is a part of her husband’s Joint Hindu family as a daughter
of that family
c. Married daughter’s right severe as member of Joint Family once she attains
majority
d. A daughter remains a part of her parents’ joint family, till the time she is
unmarried

Q76 Minakshi a 35-year-old woman was forced out of her husband’s home by his
family along with her 5-year-old son Jatin. They started living in Minakshi’s
parent's home after that. When Jatin became major he demanded a share in his
maternal grandfather’s property because his maternal; uncles refused to maintain
him on the death of his mother. Here-

35
a. Jatin is part of his maternal Joint Family and can claim his share in the property
b. Jatin is part of his maternal Joint Family but he cannot claim his share in the
property
c. Jatin is part of his Father’s joint family and must ask for his share from his own
Joint Family
d. None of the above

Q77 If XX and YY are brothers, and ZZ and QQ are XX and YY wives respectively.
XX and YY died in a car accident, ZZ and QQ can carry on the Joint Hindu family
in which of the following situation?

a. If either of them is pregnant with their husband's child


b. If both of them are pregnant it is irrelevant the child must be of the husband but
they cannot choose to adopt a child
c. If both of their contracts to be legal members of the Joint Hindu Family of their
husbands
d. Only if the husband of either widow gave them the authority to continue as
members of his Joint Hindu Family through the testamentary document

Q78 Twin brothers got married to twin sisters. After the marriage, one couple
gave birth to a daughter and one couple gave birth to a son. In an unfortunate
incident, all the males die in a car crash. Now, whether in the absence of any male
member, the family still subsists under the title of Hindu Joint Family.

a. The Hindu Joint Family is a patriarchal form of the family hence, without any
male member a family cannot call themselves a Hindu Joint Family
b. A joint Hindu family can continue to exist
c. The only daughter is part of the Hindu Joint family, not the widow
d. Nobody can be said to be part of a Hindu Joint Family here because widows can
remarry and even a daughter on her marriage becomes part of another family

Q79 Hindu law has two main schools:

a. Mitakshara school and Dayabhaga school


b. Mitakshara school and Mithila school
c. Dayabhaga school and Benaras school
d. Dayabhaga school and Dravida school
______________________________________________________________________

36
Passage 6
Read the following passage and answer the questions

Volenti non fit injuria-It basically means the voluntary assumption of risk. When a person
consents to the infliction of harm upon himself, he has no remedy for that in tort making
this an excellent defence for the defendant against tortuous liability. Consent forms an
essential part of this doctrine- whether it is implied or expressed. It must not be obtained
fraudulently (as held in R. v. Williams).
This doctrine is based on the idea that “no man can enforce a right that he himself has
waived or voluntarily abandoned”. However, the harm caused must not be beyond what
is consented to. In case, a plaintiff voluntarily suffers some harm, he has no remedy for
that under the law of tort and he is not allowed to complain about the same. The reason
behind this defence is that no one can enforce a right that he has voluntarily abandoned
or waived. Consent to suffer harm can be express or implied.

The consent must be free, For this defence to be available it is important to show that
the consent of the plaintiff was freely given. Consent obtained by fraud is not real
consent and does not serve as a good defence. Consent obtained under compulsion
There is no consent when someone consents to an act without free will or under some
compulsion. It is also applicable in cases where the person giving consent does not
have full freedom to decide. This situation generally arises in a master-servant
relationship where the servant is compelled to do everything that his master asks him to
do. The defense of Volenti non fit injuria is also not applicable in cases of negligence as
the basic constituent of the doctrine is consent- whether implied or expressed. But, if
due to some act of the defendant, the plaintiff is not left with ample time to choose to
provide consent or not, there can be no agreement to suffer harm from the said act.
Source: https://blog.ipleaders.in/general-defences-under-law-of-torts/

Q80 Rishi took his car to fill the petrol in it. Two strangers named Neeraj and Ali
took a lift in the car. The car got toppled due to some problem with the wheel. The
Neeraj and Ali fell out of the car and they suffered some injuries leading to the
death of Ali. What is the liability of Rishi?

a. He is liable for murdering Ali


b. No liability arises because they voluntarily took the lift
c. No liability arises because it was merely an accident
d. He is liable for committing grievous hurt to Neeraj and the death of Ali

37
Q81 Abha a renowned academician recently got a lot of public attention on social
media. A famous production house offered Abha money to give them the right to
document her life. When the show got aired Abha sued the production house for
revealing some parts of her life which caused her trouble. Here-

a. Abha cannot sue the production house


b. Abha can sue the production house if she did not consent to a specific part being
aired
c. Abha can only file a criminal case of defamation
d. Abha shouldn’t have agreed to get herself documented in the first place

Q82 Rajani a 54-year-old woman complained of sharp pain in her chest area and
also noticed woman noticed a lump in her breast. After the medical treatment, she
got to know that her uterus has been removed. Here-

a. The hospital authorities committed a breach of trust


b. The hospital authorities are liable because Rajani did not consent to remove her
uterus
c. The hospital authorities cannot be held liable because it is covered by the
doctrine of volenti fit injuria
d. Hospital authorities are not liable because Rajani is 54 years old and she cannot
conceive at this age so no injury is caused to her

Q83 Stephy is the plaintiff her paramour Jojo had infected her with a venereal
disease. Stephy brought a suit against him.

a. Jojo is not liable because he did not commit any fraud


b. Jojo is liable for committing an immoral act
c. Stephy cannot sue Jojo as she was not chaste
d. The action will survive

Q84 Pratik worked on a drill to cut mountain rocks. Stones were being carried
using a crane above his head. While Pratik was working suddenly a stone fell on
his head causing head injuries.

a. Crane driver was negligent as they did not inform him.


b. Crane driver is not liable because Pratik has knowledge of risk.
c. The maxim volenti non fit injuria applies in the present case.
d. Crane driver is liable only if Pratik ignores the safety instructions but still the
maxim volenti non fit injuria doesn’t apply.

38
Section - Logical Reasoning

Passage: 1

Read the passage given below and answer the questions that follow.

To avoid the worst impacts of climate change, we have to make two big transitions at
once: First, we have to generate all of our electricity from clean sources, like wind
turbines and solar panels, rather than power plants that run on coal and methane gas.
Second, we have to retool nearly everything else that burns oil and gas — like cars,
buses and furnaces that heat buildings — to run on that clean electricity. These
changes are underway, but their speed and ultimate success depend greatly on one
kind of company: the utilities that have monopolies to sell us electricity and gas. But
around the country, utility companies are using their outsize political power to slow down
the clean energy transition, and they are probably using your money to do it. State
regulators are supposed to make sure that customers’ monthly utility bills cover only the
cost of delivering electricity or gas and to set limits on how much utilities can profit. But
large investor-owned utilities, with legions of lawyers to help them evade scrutiny, bake
many of their political costs into rates right alongside their investments in electrical poles
and wires. In doing so, they are conscripting their customers into an unknowing army of
millions of small-dollar donors to prolong the era of dirty energy. The reforms are crucial
because while all corporations in the United States can spend money on politics, in
most cases, consumers who don’t approve can take their business elsewhere. Utilities
— as regulated monopolies — have the unique ability to force customers to participate.
It’s not that utilities aren’t interested in building and profiting from clean energy. Many
are doing so, and the Inflation Reduction Act offers utilities extensive tax incentives to
increase their investments in wind, solar and batteries. But that does not mean that
utilities want others to do the same. They will support a clean energy transition only if it
happens exclusively on their terms and at their pace — a stance at odds with the scope
and urgency of the herculean task of decarbonizing our electric grid. Most electric
utilities view distributed energy — technologies owned by customers that generate
electricity in smaller amounts — as a threat to their business. They have tried for years
to stop their customers in many states from investing in rooftop solar by rigging rates to
make it less economically attractive. They’ve also funded opposition to policies that
would speed clean energy.

Q85 What can be inferred conveniently from the passage?

a. Reduction Act has to be redesigned to offer more to utility companies.


b. Energy markets are vulnerable to manipulative pricing.
c. The two big transitions are enough to reverse climate change
d. Many customers have switched to homemade electricity options.

39
Q86 Which of the following, if true, would provide strength to the argument of the
author against utility companies

a. Utility companies fund the election campaign of candidates that oppose cleaner
energy
b. Government lacks the willpower to deviate from coal-based electricity.
c. No research is going on to make renewable energy cheap.
d. Customers do not understand the negative manipulative strategy of utility
companies.

Q87. Which of the following, if true, would weaken the claims of the author
against the utility companies?

a. Utility companies spend more money on research on clean energy than negative
campaigns.
b. Most of the candidates supported by utility companies have lost the recent
elections.
c. Customers are not willing to deviate from cheap dirty energy to expensive
cleaner energy.
d. The technology to store renewable energy is not effective enough.

Q88. Which of the following paradox is present in the argument of the author?

a. Government projects schemes like the ‘Reduction Act’ but officials are corrupt.
b. Utility companies block the growth of clean energy while spending on its
research.
c. Customers continue to buy dirty energy understanding the implications of it.
d. Energy companies are blamed not the government for letting them play dirty.

Q89 Which of the following faults do you find in the argument of the author?

a. He has not supported his claims with the data and facts.
b. He undermines the fact retooling everything is not feasible.
c. He does not appreciate the fact that cost is the major factor for customers.
d. He has not provided the rationale for the approach of the utility companies.

Q90. Which scenario parallels the situation in the passage where utility
companies hinder the clean energy transition?

a. A pharmaceutical company funds research for a cure but lobbies against


affordable access to existing treatments.
b. A tech company develops innovative gadgets but offers discounts on older, less
eco-friendly models.

40
c. A restaurant promotes plant-based options but increases prices for these dishes.
d. A fashion retailer introduces a sustainable clothing line but follows harmful
production practices.

______________________________________________________________________

Passage 2

Read the passage given below and answer the questions that follow.

Around 200 million people are experiencing acute food insecurity. They include some in
Afghanistan, Burkina Faso, Ethiopia, Mali, Sudan and Syria, countries that have
something else in common: each is experiencing a deadly conflict. These two situations
— hunger and conflict — are connected. In a report presented to the United Nations in
March, the organization’s special rapporteur on the right to food, Michael Fakhri, said
that violence and conflict are in fact the primary causes of hunger worldwide. They are
also pivotal reasons that the world is not on track to end hunger and malnutrition by
2030, a promise made by world leaders at a UN summit in 2015, as part of the
Sustainable Development Goals (SDGs). This is alarming for a number of reasons. For
one thing, it suggests that, unless something is done, we are abandoning hundreds of
millions of people to severe hunger. Furthermore, crucial efforts to study and implement
policies to end hunger are hampered when violence breaks out. In September, heads of
government will meet in New York City to work out what can be done. Although the
meeting takes place at a time of great tension between world powers, attendees must
accept that the SDG to end hunger will not be met unless violence is reduced — or, at
the very least, unless parties to conflict stop weaponizing food. Fakhri’s report draws on
decades of studies, as well as newer data from bodies including the UN’s World Food
Programme and the Food and Agriculture Organization of the UN. The report describes
the relationship between violence in various forms, including sexual and gender-based
violence, and food insecurity. Conflicts endanger food security when, for example, crops
are destroyed or food supplies are disrupted — things that have happened, and
continue to happen, in wars from Mali to Myanmar. Coercive measures, such as
international economic sanctions against warring countries, also contribute to hunger.
The evidence, according to the report, is that ‘targeted’ sanctions disrupt food systems,
too.

Q91 What statement weakens the argument that violence is the sole reason the
world is not on track to end hunger by 2030?

a. The number of violence hit countries is very less.


b. The agricultural output has reduced over time.
c. There is enough food to sustain the current population.

41
d. Food production is highly concentrated only in a few countries.

Q92 What can be inferred from the passage?

a. The upcoming meeting of heads of government will focus on reducing violence


b. International economic sanctions effectively address food insecurity in
conflict-affected countries.
c. Violence and conflict disproportionately affect vulnerable populations.
d. The report's findings will lead to immediate policy changes.

Q93. Which of the following is the paradox that the passage leaves the reader
with?

a. Food Insecurity is not related to food production and distribution but to violence
b. War leads to disruption in food supply but sanctions on warring nations are not
recommended either.
c. The countries with the problem of food insecurities mentioned here are
economically weak as well.
d. None of these

Q94. Which of the following option best describes the purpose of ‘weaponizing
food’ as interpreted in the passage?

a. Manipulating or controlling the production, distribution, or access to food.


b. A means of exerting power, gaining leverage, or influencing outcomes in a
conflict.
c. Using food as a strategic tool to arm-twist international organizations like United
Nations.
d. Establishing pressure on the warring countries to inflict sanctions.

Q95. Which of the following can be concluded about the ‘Sustainable


Development Goals’?

a. There is a lack of due consideration in deciding them.


b. These are not taken seriously by the countries at war.
c. There is not much effort being applied to achieve them.
d. None of these

42
Q96. Based on the passage, which of the following can be inferred about the
relationship between violence and achieving the Sustainable Development Goal
(SDG) to end hunger and malnutrition by 2030?

a. Achieving the SDG to end hunger and malnutrition by 2030 is possible without
addressing violence and conflict.
b. The presence of violence and conflict significantly hinders progress towards
achieving the SDG to end hunger and malnutrition by 2030.
c. The SDG to end hunger and malnutrition by 2030 does not prioritize addressing
the impact of violence and conflict on food insecurity.
d. The relationship between violence and achieving the SDG to end hunger and
malnutrition by 2030 is not clearly established in the passage.

______________________________________________________________________

Passage 3

Read the passage given below to answer the questions that follow-

The creative work of the human mind is protected through several measures and the
main motivation for the same is that such protection is a definite measure of
encouragement for the creative activity. Several forms of protection of the creative
activity have come about including those which are of particular interest in industrial
development. Patents being one of them. Generally speaking, the patent is a monopoly
grant and it enables the inventor to control the output and within the limits set by
demand, the price of the patented products. The underlying economic and commercial
justification for the patent system is that it acts as a stimulus to investment in Industrial
innovation. The grant of the first patent can be traced as far back as 500 B.C. It was the
city dominated by gourmands, and perhaps the first, to grant what we nowadays call
patent right to promote culinary art. For it conferred exclusive rights of sale to any
confectioner who first invented a delicious dish. As the practice was extended to other
Greek cities and other crafts and commodities, it acquired the name ‘monopoly’, a
Greek Portmanteau word from mono (alone) and polein (sale). Evidence of grants to
private individuals by kings and rulers of exclusive property rights to inventors dates
back to the 14th Century, but their purpose varied throughout history. History shows that
in the 15th Century in Venice, there had been systematic use of monopoly privileges for
inventors for the encouragement of invention. The utility and novelty of the invention
were the important considerations for granting a patent privilege. The inventors were
also required to put their inventions to commercial use within a specified period. In the
16th Century, the German princes awarded inventors of new arts and machines and
also took into consideration the utility and novelty of inventions.

43
Q97 What is the main purpose of the author in the passage?

a. He is looking to highlight the fact that patents can lead to a monopoly.


b. The author is describing the benefits of patents to avoid monopolistic markets.
c. The passage is an attempt to connect with the history of patents.
d. It is to give readers an insight into the needs of patents.

Q98. Based on the facts given in the passage, what can be inferred?

a. People of different eras find patents useful to an extent only.


b. Different eras considered monopoly privileges as patents.
c. The concept of patent is a generation of business society.
d. The design of patents has changed but the objective remains the same.

Q99. With reference to the invention, it can be logically assumed that –

a. Patents imbibe creativity in artists and inventors.


b. Knowledge of the rules of patents can lead to encouraging creativity.
c. Creativity and inventions can be saved only by patenting.
d. Patents lead to a monopoly that kills inventions in the long term.

Q100. Which of the following statement strengthens the argument propagated by


the author in favour of patents?

a. Patent can provide an important source of revenue for your business.


b. Patent means making certain technical information about your invention publicly
available.
c. Taking action against an infringer can be a very expensive process.
d. You can believe your invention has potential in other countries’ markets as well.

Q101 For which of the following patents could be applied as per the passage-

a. Discovery of a scientific principle


b. An invention whose primary goal or intended use is in accordance with the law.
c. Any invention that might be frivolous or trivial.
d. An invention which in effect is traditional knowledge.

44
Q102. Which of the following assumptions underlies the argument presented in
the passage about the economic and commercial justification for the patent
system?

a. The grant of exclusive rights through patents encourages inventors to invest in


industrial innovation.
b. All forms of creative work are eligible for patent protection.
c. Patents are primarily granted to protect the interests of large corporations and
businesses.
d. The patent system is the only means of protecting intellectual property and
encouraging creativity.

______________________________________________________________________

Passage 4

Read the passage given below and answer the questions that follow.

Marx’s theory provides the best theory by far of profit, the all-important dominant
purpose of capitalist production, and the main question in a theory of capitalism.
Mainstream economic theories provide almost no theory of profit at all, and certainly no
theory with comparable explanatory power to Marx’s. Mainstream macroeconomic
theory has no theory of profit at all; in other words, mainstream macroeconomics
attempts to provide a theory of capitalism without a theory of profit! It is like trying to
have a theory of the Catholic church without the Pope. Mainstream microeconomics has
attempted to provide a theory of profit (or what it calls interest), the “marginal
productivity” theory, according to which interest is determined by the marginal
productivity of capital. However, this theory has been shown to be logically contradictory
(as a result of the “capital controversy”) and has little or no explanatory power. This
theory is now in general disrepute and is being quietly dropped (in hopes that no one
will notice) from microeconomics textbooks at both the graduate and undergraduate
levels. Marx’s theory, by striking contrast, provides a logically robust theory of profit with
very impressive explanatory power. Marx’s theory of profit is of course that profit is
produced by the surplus labor of workers. That is, it only takes a part of the working day
for workers to produce a value equal to their wages (the “necessary labour” portion of
the working day). In the remainder of the working day, the value produced by workers
becomes the profit of capitalists. Therefore, Marx’s theory concludes that the profit of
capitalists is the result of the exploitation of workers because the value produced by
workers is greater than the wages they are paid. It follows from this theory that
capitalism is inherently and unavoidably an unjust and exploitative economic system.
Capitalism cannot exist without profit, and profit cannot exist without the exploitation of
workers. No number of reforms within capitalism can alter this basic fundamental truth.

45
If we want a just and equitable economic system without exploitation, then Marx’s
theory suggests that we must change the economic system from capitalism to
socialism.

Q103.What argument does the author present against the "marginal productivity"
theory in microeconomics?

a. The theory contradicts the concept of profit.


b. The theory lacks explanatory power.
c. The theory has been discredited by the "capital controversy."
d. The theory is being dropped from microeconomics textbooks.

Q104. Which of the following, if true, would weaken the author's argument that
Marx's theory of profit is superior to mainstream economic theories?

a. Mainstream microeconomics has recently developed a new theory of profit that


addresses the flaws in the "marginal productivity" theory.
b. Marx's theory of profit fails to account for the role of technological advancements
in capitalist production.
c. Mainstream macroeconomic theories have been successful in predicting
economic trends and outcomes.
d. Leading economists from various schools of thought have criticized Marx's theory
of profit for its lack of empirical evidence. give an answer and explanation and
reason for incorrect options in brief

Q105. Which of the following options provides the most logical way for
mainstream theories to return to the textbooks?

a. The flaws in mainstream theories are addressed and resolved through rigorous
academic research and scholarly debates.
b. Mainstream economics textbooks include a separate section discussing
alternative theories, including Marx's theory of profit.
c. Leading economists advocate for the inclusion of mainstream theories in
textbooks to provide a comprehensive understanding of economic principles.
d. Mainstream economics departments collaborate with experts in Marxian
economics to integrate both perspectives into their curricula.

Q106.Which of the following can be inferred about capitalism and the state of
workers based on the passage?

a. Many workers in capitalist economies face income inequality and struggle to


make ends meet.

46
b. In capitalist societies, workers though overworked are rewarded based on their
skills and qualifications.
c. The exploitation of workers is prevalent in all capitalist countries, regardless of
their economic development.
d. Capitalistic economies have the necessity of comprehensive social welfare
programs for labourers.

Q107 Which of the following assumption follows from the passage about
socialism?

a. Socialism is a utopian economic system with no grievances of labours.


b. Socialism provides the economic system with the potential to minimise
exploitation.
c. Unlike capitalism, socialism ensures profits and social equality as well.
d. Socialism might have its drawbacks but it is better than capitalism.

Q108. Based on the information provided in the passage, which of the following
can be inferred about the role of patents in encouraging innovation and
investment in industrial development?

a. Patents provide legal protection for creative work, leading to increased


investment in industrial innovation.
b. The patent system is the sole means of encouraging creativity and investment in
industrial development.
c. Mainstream economic theories acknowledge the superior role of patents in
stimulating innovation.
d. Marx's theory of profit offers a more robust explanation for the role of patents in
industrial development.

______________________________________________________________________
Section - Quantitative Techniques
Passage: 1
Read the following information carefully and answer the multiple-choice
questions that follow.

A company conducted a survey to determine the percentage of its employees who have
different levels of education. The results of the survey are as follows
- Total number of employees surveyed: 800
- Percentage of employees with a high school diploma: 40%

47
- Percentage of employees with a bachelor's degree: 30%
- Percentage of employees with a master's degree: 20%
- Percentage of employees with a Ph.D.: 10%

Q109 How many employees surveyed have a bachelor's degree?


a. 200
b. 240
c. 260
d. 280

Q110: What percentage of employees surveyed have at least a master's degree?


a. 20%
b. 30%
c. 40%
d. 50%

Q111: If the total number of employees surveyed is 1200 instead of 800, what
would be the ratio of number of employees with a high school diploma originally
to updated?
a. 1 : 3
b. 2 : 5
c. 2 : 3
d. 2 : 7

Q112: If 20% of the employees with a Ph.D. are women, and 40% of the employees
with a high school diploma are women, what is the minimum percentage of
women among all the employees surveyed?
a. 28%
b. 32%
c. 18%
d. 40%

48
Passage: 2

Read the passage given below and answer the questions that follow.

Directions: Read the following information carefully and answer the multiple-choice
questions that follow.
In the heart of Pune commercial district, lies the bustling marketplace known as Balaji.
Balaji hosts a plethora of shops offering a diverse range of products. From apparel to
electronics, shoppers are spoiled for choice. Amidst the hustle and bustle, some shrewd
business owners are engaging in complex profit and loss calculations to strategize their
pricing and maximize their gains.
Scenario: Clothing Store's Inventory
Seema, the owner of a high-end clothing store named Seema’s clothing house, is
assessing her inventory at the end of the season. Here are the costs (in Rs) and selling
prices of five premium dresses:
1. Dress A: Cost - Rs200, Selling Price - Rs350
2. Dress B: Cost - Rs180, Selling Price - Rs290
3. Dress C: Cost - Rs220, Selling Price - Rs340
4. Dress D: Cost - Rs250, Selling Price - Rs360
5. Dress E: Cost - Rs190, Selling Price - Rs300
Seema aims to calculate the total profit or loss on her dress inventory.
Scenario: Electronics Store's Markdown Strategy
ElectroTech, a prominent electronics store in Balaji, is executing a markdown strategy to
clear out some of their older stock. They have reduced the prices of four electronic
gadgets:
1. Gadget X: Original Price - Rs480, Markdown Price - Rs300
2. Gadget Y: Original Price - Rs560, Markdown Price - Rs380
3. Gadget Z: Original Price - Rs400, Markdown Price - Rs240
4. Gadget W: Original Price - Rs620, Markdown Price - Rs450
ElectroTech now needs to determine the total percentage decrease in revenue due to
these markdowns.

49
Scenario: Grocery Store's Bulk Purchase
At Balaji’s GroceryGalore, the store manager, Mrs. Anjani, seizes an opportunity for a
bulk purchase of essential commodities. He buys a large quantity of rice at a discounted
rate:
1. Quantity Purchased: 500 kg
2. Original Rate: Rs2.50 per kg
3. Discounted Rate: Rs2.00 per kg
Mrs. Anjani is eager to ascertain the total savings due to this bulk purchase.
Scenario: Tech Accessories Store's Loss
TechHaven, a specialized store in selling tech accessories, encountered a loss during a
promotional event. They offered a discount on various items, including the following:
1. Accessory P: Cost - Rs30, Selling Price - Rs20
2. Accessory Q: Cost - Rs40, Selling Price - Rs30
3. Accessory R: Cost - Rs50, Selling Price - Rs40
TechHaven is now calculating the total loss incurred during the promotional event.
Scenario: Electronics Store's Bundle Offer
ElectroZone, a prominent electronics store in Rodo, has introduced a special bundle
offer to attract more customers. The bundle includes two gadgets and a tech accessory,
each with its original price and the discounted bundle price:
1. Gadget A: Original Price - Rs480
2. Gadget B: Original Price - Rs550
3. Tech Accessory X: Original Price - Rs40
Bundle Price: Rs980
ElectroZone aims to calculate the total percentage discount offered in the bundle deal.

Q113- What is the total profit or loss on the dress inventory of Fashion
Emporium?
a. Rs 600 profit
b. Rs 670 loss
c. Rs 710 profit
d. Rs 710 loss

50
Q114. What is the total percentage decrease in revenue for ElectroTech after the
markdown strategy?
a. 26.47%
b. 28.12%
c. 33.50%
d. 32.56%

Q115- What is the total savings made by GroceryGalore on the bulk purchase of
rice?
a. Rs 250
b. Rs 275
c. Rs 300
d. Rs 325

Q116- What is the total loss incurred by TechHaven during the promotional event?
a. Rs 25
b. Rs 30
c. Rs 35
d. Rs 40
______________________________________________________________________
Passage 3
Read the following information carefully and answer the multiple-choice
questions that follow.

In a bustling classroom of 40 bright young minds, the students have been working hard
to excel in their studies. Their dedication and hard work have paid off, as evidenced by
their impressive average marks in different subjects. In Mathematics, the class has
shown a strong aptitude for numbers and problem-solving, achieving an average mark
of 85. In Science, the students have demonstrated a keen understanding of the natural
world, with an average mark of 75. The class has also excelled in the language arts,

51
with an average mark of 90 in English, showcasing their mastery of grammar and
composition. In History, the students have delved into the past and gained a deeper
understanding of the events that have shaped our world, achieving an average mark of
80. These remarkable young students continue to strive for excellence in all their
academic pursuits.
Let's analyze the data to answer some questions related to average marks.

Q117: If the average marks of the students in all four subjects are considered,
what is the overall average mark of the class?
a. 82.5
b. 80.5
c. 82.5
d. 92.5

Q118: If the class average mark in Mathematics increases by 5 points and the
class average mark in Science decreases by 5 points, what would be the new
overall average mark of the class in these four subjects?
a. 70.5
b. 79.5
c. 82.5
d. 80.5
Q119: If the class wants to maintain an average mark of 80 across all subjects,
what is the minimum average mark they need to achieve in another subject?
a. 70
b. 75
c. 80
d. 85

Q120: If the average marks of the class are increased by 5 in each subject, what
will be the new overall average mark of the class?
a. 87.5

52
b. 85.5
c. 84.5
d. 83.5

______________________________________________________________________

53
Answer Key
__________________________________________

1 2 3 4 5 6 7 8 9 10 11 12 13 14 15
b c a c c a b d d a b b b b c
16 17 18 19 20 21 22 23 24 25 26 27 28 29 30
b b a c a a c c c c c a d c a
31 32 33 34 35 36 37 38 39 40 41 42 43 44 45
b c c b d c c b c a b a a d c
46 47 48 49 50 51 52 53 54 55 56 57 58 59 60
c b b a b c a b a b b a d b c
61 62 63 64 65 66 67 68 69 70 71 72 73 74 75
d d b c a d a d b d a a a c d
76 77 78 79 80 81 82 83 84 85 86 87 88 89 90
c a b a c b b a a b a b b a c
91 92 93 94 95 96 97 98 99 100 101 102 103 104 105
b c b b d b c d b a b a c d a
106 107 108 109 110 111 112 113 114 115 116 117 118 119 120
a b a b b c c a c a b c c a a

______________________________________________________________________

54
Solutions
Section - English

1-b
According to the author, emerging AI technologies do not replace or suppress creativity.
Instead, the technology aims to be considered an advanced state of creativity by
connecting to human art history. The author has cited the example of “The Rise of
Metacreativity: AI Aesthetics After Remix” by Eduardo Navas, published in 2022, in this
context.
Explanation of incorrect option
Option A - Incorrect
This option is not supported by the passage. The passage discusses how AI
technologies, particularly in the context of art, have been used to explore new forms of
creativity, such as remixing and modularity. It does not suggest that AI technologies are
replacing creativity entirely. Instead, it highlights how AI can be seen as an advanced
level of creativity, complementing human creativity rather than replacing it.
Option C - Incorrect
This option is incorrect. AI and art are indeed discussed as two sides of a coin in the
passage, with AI offering new avenues and techniques for artistic expression and
creativity but option B says the exact thing.
Option D - Incorrect
The passage clearly presents the view that AI technologies represent an advanced level
of creativity and do not suppress creativity but rather expand artistic possibilities.

2-c
According to the author, linking an artwork with an artist’s technical skills is a myth. It is
because the ancient cave paintings depict the communal rituals of the people prevailing
at that time. Eastern art is an example of imitating the art of past artists to preserve the
continuity of the style.
Explanation of incorrect options:

55
Option A - Incorrect
This option is not stated as a myth in the passage. The passage discusses how art,
including cave paintings and Eastern art, has been linked to communal rituals. It does
not label this association as a myth but rather provides examples of art being connected
to communal practices.
Option B - Incorrect
This option is not presented as a myth in the passage. Instead, the passage highlights
various art practices throughout history, such as collage, readymades, and pop art,
where artists have imitated and remixed existing styles as a valid form of artistic
expression.
Option D - Incorrect
This option is not discussed as a myth in the passage. The passage focuses on various
aspects of art, such as chance, modularity, and remix, but it does not specifically
address the idea that art is meant to highlight the artist's personality as a myth.

3-a
Incapable of being broken
Explanation of incorrect option:
Option B - Incorrect
Not compatible - Inconsistent
Option C - Incorrect
The quality of being honest - Integrity
Option D - Incorrect
Without interest or concern – Indifferent

4-c
Art is tied to three major components, namely, chance, remix, and modularity. Modules
are the pre-existing art materials which are combined to create collage art. In this
context, the rest of the options are not correct.
Explanation of incorrect option
Option A - Incorrect

56
The passage does mention chance as a component of art, but it is not specifically
connected to pop art in the context of the given passage. The passage discusses
chance as a "meta-algorithm" that has been employed by various artists, including
Dadaists, surrealists, and futurists, and later by artists like John Cage and Nam June
Paik. However, there is no direct association made between chance and pop art in the
passage.
Option B- Incorrect
The passage does discuss remix as a component of art, which is borrowed from the
sphere of music and applied to the realm of images and texts. However, there is no
mention of readymades in the passage, so there is no direct connection between
remixesremix and readymade art.
Option D - Incorrect
The passage does not mention algorithms as a component of art. While it does discuss
the concept of "meta-algorithm" related to chance, it does not specifically refer to
algorithms in connection with art. The passage also mentions "action painting" as a
practice that has unhinged art from the artist's labor and technique, but there is no direct
link between algorithms and action painting in the passage.

5-c
The given statement refers to the situation that the AERB is compliant with the DAE. It
is because India, as a signatory of the Convention on Nuclear Security, should abide by
the rule that the nuclear regulator of a country should be independent which means it
should be separate from any other organisation which promotes or utilises nuclear
energy.
Explanation of incorrect option:
Option A - Incorrect
The passage mentions the establishment of the NPCIL as a public company to design,
construct, and operate nuclear power plants under the administrative control of the
DAE, but it does not provide any further details or discussion about its implications or
relevance to India's promises to the global community.
Option B - Incorrect

57
This option is not elaborated on in the passage. The passage mentions the
establishment of the DAE in 1954 with the goal of producing nuclear-powered electricity
for India and developing nuclear technology. However, it does not provide any further
details or discussion about the establishment of the DAE under the Atomic Energy Act,
of 1962, and its connection to India's promises to the global community.
Option D - Incorrect

The passage mentions that the DAE proposed the Nuclear Safety Regulatory Authority
Bill after the Fukushima disaster, which suggested dissolving the AERB and
establishing a more independent regulator. However, the passage does not provide any
further details or discussion about the proposal and its implications or relevance to
India's promises to the global community.

6-a
“Subservience” means a willingness to serve or obey others’ instructions without any
objection. A person who is obsequious in behaviour is subservient.
Option B - Incorrect
This option gives the exact opposite of the word "subservience." "Subservience" implies
being obedient and compliant, rather than refusing to obey. Therefore, this option is not
the correct description of "subservience."
Option C - Incorrect
This option does not accurately describe the meaning of "subservience." "Readiness to
be engaged in a difficult activity" does not capture the idea of being obedient or
compliant, which is the essence of "subservience."
Option D - Incorrect
This option does not describe the meaning of "subservience." "A lack of willingness to
do something" suggests reluctance or unwillingness, while "subservience" implies being
obedient and compliant. Therefore, this option is not the correct description of
"subservience."

58
7-b
According to the last paragraph of the passage, the Fukushima disaster stimulated the
DAE to propose the Nuclear Safety Regulatory Authority Bill about the dissolving of the
AERB and the establishing of an independent regulator. As a result, this suggestion was
examined and the required changes were made.
Explanation of incorrect options:
Option A - Incorrect
The last paragraph mentions that the DAE proposed the Nuclear Safety Regulatory
Authority Bill after the Fukushima disaster, but it does not state that the proposal was
about the DAE becoming an independent body under the NPCIL. The focus of the
proposal was to dissolve the AERB and establish a more independent regulator, not to
change the status of the DAE.
Option C - Incorrect
This option is not supported by the last paragraph of the passage. The passage
mentions the establishment of the AERB as a 'watchdog' body, meant to regulate and
enforce safety norms for all nuclear activities, including those of the DAE. The passage
also mentions that the AERB reports to the Atomic Energy Commission (AEC), which is
headed by the secretary of the DAE. However, there is no mention of the DAE having
the power to appoint a subordinate authority for supervising nuclear authorities.
Option D- Incorrect
This option is not supported by the last paragraph of the passage. The last paragraph
does not mention any violation of the Atomic Energy Act after the setup of the AERB
under the AEC. Instead, it discusses how the AERB's compliance with the DAE as a
regulating body of nuclear activities puts India at odds with its promises to the global
community, particularly with regard to the Convention on Nuclear Security.

8-d
The central idea of the passage revolves around the conflict between India's promises
to the global community regarding nuclear safety and the compliance of the Atomic
Energy Regulatory Board (AERB) to the Department of Atomic Energy (DAE). The
passage discusses how India was one of the first signatories of the Convention on

59
Nuclear Security, which requires a country's nuclear regulator to be effectively separate
from any other body or organization that promotes or utilizes nuclear energy. However,
the passage highlights that the AERB, which is meant to be the nuclear watchdog,
reports to the Atomic Energy Commission (AEC), which is headed by the secretary of
the DAE.
Explanation of incorrect options:
Option A - Incorrect
This option is too broad and does not capture the specific conflict discussed in the
passage. While the passage briefly mentions the history and development of nuclear
energy in India, it primarily focuses on the conflict between India's promises to the
global community and the compliance of the AERB with the DAE.
Option B - Incorrect
This option is not the central idea of the passage. The passage does mention the
establishment of the DAE and its goals, which include producing nuclear-powered
electricity for India and developing nuclear technology. However, this information is
provided as background context rather than the main focus of the passage.
Option C - Incorrect
This option is partially related to the central idea of the passage but does not
encompass the full conflict discussed. The passage does discuss the role of the AERB
as a regulatory body meant to supervise nuclear activities, but the central idea is more
about the conflict arising from the AERB's compliance with the DAE, which affects its
independence as a regulator. Therefore, this option is not the central idea of the
passage.

9-d
According to the second paragraph of the passage, the launching of Chandrayaan-3 is
the second attempt of India to land a lander with a rover on the moon’s surface.
Explanation of incorrect option
Option A - Incorrect
The second paragraph mentions that soft landingsoft-landing on the moon is a
complicated exercise, and the possibility of failure exists. It also discusses the

60
consequences of complete success if the mission succeeds. Therefore, the passage
acknowledges that there is a possibility of failure in the Chandrayaan-3 mission, and
this option is not correct.
Option B - Incorrect
The second paragraph mentions that Chandrayaan-2's surface mission failed in
September when the lander crashed, and the failure was identified to be due to a
problem in the guidance software and unexpected dispersion in the propulsion system
during certain phases of the descent. It does not mention any issues with the propulsion
system while ascending, making this option incorrect.
Option C - Incorrect
The paragraph mentions that the lander of Chandrayaan-3 will begin a series of
maneuvers culminating in a gradual landing over the surface on August 23-24, 2023. It
does not specify the exact time frame from the day of launching, making this option
incorrect.

10 - a
The given paragraph is an example of a scientific non-fiction text. It is neither a short
story nor a research paper. Also, it is not fiction because it is based on real facts.
Explanation of incorrect options
Option B - Incorrect
Fiction refers to imaginative or invented stories, characters, and events that are not
based on real facts or events. The passage provided is not a work of fiction; it presents
factual information about the Chandrayaan-3 mission and India's space exploration
efforts. Therefore, this option is not correct.
Option C - Incorrect
A short story is a fictional prose narrative that typically focuses on a single event,
character, or theme. The passage is not a short story but rather a factual piece of writing
providing information about the Chandrayaan-3 mission and related developments in
India's space research. Therefore, this option is not correct.
Option D - Incorrect

61
A research paper is a formal academic document presenting the author's research
findings, analysis, and conclusions on a specific topic. While the passage provides
information about the Chandrayaan-3 mission, it is not structured as a formal research
paper presenting research findings and analysis. Instead, it is a factual piece of writing
presenting information about the upcoming mission and related aspects. Therefore, this
option is not correct.

11 - b
The passage says that there have been significant upgrades in the physical as well as
technical aspects of the lander to save it from crashing as was the case for
Chandrayaan-2. So, it is the main idea of the underlined part.
Explanation of incorrect options
Option A - Incorrect
This option is not supported by the passage. While the passage mentions that
Chandrayaan-3 is largely a replica of its predecessor, Chandrayaan-2, and will attempt
to soft-land a lander and rover on the moon, it does not explicitly state that
Chandrayaan-3 is the successor of Chandrayaan-2. The term "successor" implies a
direct follow-up, which is not mentioned in the passage. Therefore, this option is not
correct.
Option C - Incorrect
The passage mentions that if the Chandrayaan-3 mission succeeds, India will be the
second country to have soft-landed a rover on the moon. While the success of the
mission may enhance India's standing in the field of moon-landing missions, the
passage does not state that India's chances of leading such missions have increased
with this launch. Therefore, this option is not correct.
Option D - Incorrect
The passage mentions that the mission has six scientific payloads to study various
features of the moon's surface, but it does not state that this is the maximum number of
scientific payloads to date. Therefore, this option is not correct.

62
12 - b
“Manoeuvre” means a series of skilled and careful movements. Some of its synonyms
are operation, activity, and movement.
Explanation of incorrect options
Option A - Incorrect
While "manoeuvre" can sometimes be used in the context of a project, in the passage, it
is used in the sense of a specific action or set of actions that the lander will perform to
navigate and position itself during the landing process. "Project" is a broader term that
may include various activities, whereas "manoeuvre" refers to a specific operation or
action.
Option C - Incorrect
"Cessation" means the act of stopping or coming to an end, which does not capture the
meaning of "manoeuvre" in the context of the passage. In the passage, "manoeuvre"
refers to the series of actions the lander will perform during the landing process, which
is not related to stopping or ending.
Option D - Incorrect
"Exertion" refers to physical or mental effort or application, which is not the intended
meaning of "manoeuvre" in the passage. In the passage, "manoeuvre" refers to the
operational actions that the lander will undertake during its landing on the moon.

13 - b
The teachers helped the author to learn various secrets of science such as the
differences between freedom and escape, between motion and movement, and
between slide and flow.
Explanation of incorrect options
Option A - Incorrect
The passage does mention the author's father's inspiring words about learning and
wisdom, but there is no specific indication that the teachers helped the author learn with
the wisdom of knowing himself. The teachers are mentioned to have created excitement
about aeronautics through their meticulous teaching and intellectual fervour, but there is
no direct connection to the author's self-knowledge in this context.

63
Option C - Incorrect
The passage mentions that around the time the author chose aeronautical engineering,
he made special efforts to try and communicate with different kinds of people. However,
there is no direct mention of the teachers' role in encouraging the author to
communicate with different types of people. The focus of the passage is more on the
author's fascination with aeronautics and the impact of his teachers' teaching on his
interest in the subject.
Option D- Incorrect
While the passage briefly mentions the author's contemplation of the relationship
between his belief in God and scientific thinking, there is no specific indication that the
teachers played a role in clarifying the conflicts between his religious and scientific
thinking. The passage mentions the author's own ponderings on the matter, and the
focus is more on his interest in aeronautics and the impact of his teachers on his
understanding of the subject.

14 - b
Learning can be positively accelerated with the guidance of intellectual educators. It can
be referred from the second paragraph of the passage that the guidance of his teachers
made him learn some significant concepts of science along with the structural features
of aeroplanes.
Explanation of incorrect options
Option A - Incorrect
The passage does mention the author's father's inspiring words about learning and
wisdom, but it does not suggest that learning without wisdom is sufficient to develop
concepts. Instead, the passage emphasizes the importance of wisdom in learning and
the value of understanding oneself in the process of learning. The passage also
discusses the author's teachers' role in creating excitement about aeronautics through
their meticulous teaching and intellectual fervour. Therefore, this option is not correct.
Option C - Incorrect
While the passage does mention the author's choice of aeronautical engineering and his
fascination with the subject, it does not make a direct statement that learning

64
aeronautical engineering is the best choice to pursue a bright career. The focus of the
passage is more on the author's interest in aeronautics and the impact of his teachers'
guidance on his learning experience, rather than making a general claim about
aeronautical engineering as the best career choice.
Option D - Incorrect
The passage briefly mentions the author's contemplation of the relationship between his
religious beliefs and scientific thinking. However, it does not make a strong claim that
learning science makes religious beliefs sound unreal. Instead, the passage presents
the author's ponderings on the matter without firmly asserting a specific conclusion. The
primary focus of the passage is on the author's fascination with aeronautics and the
impact of his teachers' guidance on his learning experience. Therefore, this option is not
correct.

15 - c
“Meticulous” means a perfectionist which is opposite in meaning to “unscrupulous”.
“Unscrupulous” means immoral.
Explanation of incorrect option
Option A - Incorrect
“Fascinating” means interesting which is similar in meaning to “enchanting.”
“Enchanting” means delightful.
Option B - Incorrect
“Nebulous” means confused which is similar in meaning to “ambiguous”. “Ambiguous”
means something that has more than one meaning.
Option D - Incorrect
“Perception” means understanding which is similar in meaning to “approach”.
“Approach” means a way to arrive at a conclusion.

16 - b
The tone of the passage is technical because it deals with the subject of science.
Explanation of incorrect options
Option A - Incorrect

65
Humorous means funny or amusing.
Option C - Incorrect
Vitriolic means full of anger.
Option D - Incorrect
Satirical means taunting.
Hence, option B is correct.

17 - b
According to the first paragraph of the passage, India is on the way to growth by
converting problems into advantages. It says that the Indian economy has recovered
from the twin-balance sheet problem of banks and corporates to the twin-balance sheet
advantage. All this has been possible due to the efforts of the Modi Government. The
profit estimation is also given to be tripled in 2022-23, as compared to the value in 2014.
Explanation of incorrect options:
Option A - Incorrect
The passage mentions that the Finance Minister, Nirmala Sitharaman, stated that the
Indian economy had moved away from the twin-balance sheet problem of banks and
corporates to twin-balance sheet advantage. It does not indicate that the Indian
government is struggling to upgrade the financial health of corporates. Instead, the
passage highlights the positive outcome of the government's initiatives, leading to an
improvement in the performance of public sector banks and a shift from the
twin-balance sheet problem to the twin-balance sheet advantage.
Option C - Incorrect
The passage does not make any direct comparison of India's financial strength with that
of other nations. While it mentions the improvement in the profit of public sector banks
and the positive impact of the government's initiatives, it does not assert that India is
one of the most financially strong nations in the world.
Option D - Incorrect
The passage mentions the 4R strategy followed by the Modi government to revitalise
state-owned banks, but it does not explicitly state that India has successfully
implemented the strategy. Instead, it highlights the improvement in the performance of

66
public sector banks on account of various initiatives taken by the Modi government
since 2014.

18 - a
According to the content of the passage, the main point of concern is that the credit
outreach in the northeastern and eastern states where the credit offtake is lower than
the national average, should be focused on by the finance departments. The aim should
be to increase brick-and-mortar banking presence in border areas, particularly in those
covered in the Vibrant Village Programme.
Explanation of incorrect options
Option B - Incorrect
While the passage does mention the 4R strategy followed by the Modi government to
revitalize state-owned banks, it does not specifically focus on its implementation in
Punjab. The primary focus of the passage is on the improvement of public sector banks'
performance, the shift from twin-balance sheet problem to twin-balance sheet
advantage, and the need to focus on credit outreach in states with lower credit offtake
than the national average. The passage mentions the Punjab and Sind Bank as an
example of successful growth, but it does not make the implementation of the 4R
strategy in Punjab the main point of concern.
Option C - Incorrect
While the passage briefly mentions the story of the Punjab and Sind Bank and its
growth from two branches in 1947 to 1,553 branches, it does not emphasize a
significant increase in the number of Punjab and Sind Banks as the main point of
concern. The passage primarily discusses the improvement of public sector banks'
performance, the shift from twin-balance sheet problem to twin-balance sheet
advantage, and the need to focus on credit outreach in states with lower credit offtake
than the national average.
Option D - Incorrect
This option is not the main point of concern in the passage. While the passage mentions
the improvement in all critical parameters like return on asset, net interest margin, and
provisioning coverage ratio of public sector banks, it does not make the improvement of

67
all parameters associated with the financial health of banks the main point of concern.
The main point of concern in the passage is related to the shift from twin-balance sheet
problem to twin-balance sheet advantage and the need to focus on credit outreach in
states with lower credit offtake than the national average.

19 - c
The tone of the passage is optimistic as it talks about overcoming the problem of the
twin-balance sheet due to the initiatives taken by the Modi government since 2014.
Explanation of incorrect options
Option A - Incorrect
“Satirical” means taunting.
Option B - Incorrect
“Provocative” means stimulating or irritating.
Option D - Incorrect
“Ethical” means dealing with principles of morality.
Other options do not justify the answer.

20 - a
“Optimum” means the most favourable level of growth. “Excellent” means extremely
good.
Eplanation of incorrect options
Option B - Incorrect
“inferior” means lower in quality.
Option C - Incorrect
“Unsatisfactory” means not good enough.
Option D - Incorrect
“Wretched” means something of a very poor quality.

68
21 - a
The passage mentions that advancements in technology, such as the availability of
smartphones and internet access, have made social media more accessible and
user-friendly. This has contributed to the exponential growth of social media.
Explanation of Incorrect Options:
b) This option contradicts the information provided in the passage, which states that the
COVID-19 pandemic accelerated the growth of social media.
c) This option contradicts the information provided in the passage, which highlights the
user-friendly nature of social media platforms as a contributing factor to their growth.
d) This option contradicts the information provided in the passage, which states that
social media has experienced global growth and is not limited to specific geographical
areas.

22 - c
The passage states that the COVID-19 pandemic accelerated the growth of social
media as people turned to these platforms to stay connected, informed, and entertained
during lockdowns and restrictions.
Explanation of Incorrect Options:
a) This option contradicts the information provided in the passage, which highlights the
accelerated growth of social media during the pandemic.
b) This option is not supported by the information provided in the passage, which
focuses on the growth and popularity of social media during the pandemic.
d) This option contradicts the information provided in the passage, which specifically
mentions the accelerated growth of social media due to the pandemic.

23 - c
The passage mentions that influencers have emerged as a powerful force on social
media, with their ability to shape opinions, trends, and consumer behaviour. This
indicates that influencers play a significant role in influencing social media users.
Explanation of Incorrect Options

69
a) This option contradicts the information provided in the passage, which highlights the
influential role played by influencers on social media.
b) This option goes beyond the scope of the passage, which does not specifically
address the role of influencers in spreading misinformation.
d) This option contradicts the information provided in the passage, which emphasises
the powerful role of influencers on social media.

24 - c
The passage mentions that social media has revolutionised communication, provided
opportunities for self-expression and entrepreneurship, but also raised concerns about
privacy, mental health, and the spread of misinformation. This indicates that the author
recognizes both the positive and negative impacts of social media on society.
Explanation of Incorrect Options
Option A - Incorrect
This option is not supported by the information provided in the passage, as the author
acknowledges the negative aspects of social media as well.
Option B - Incorrect
This option is not supported by the information provided in the passage, as the author
acknowledges the positive aspects of social media as well.
Option D - Incorrect
This option is contradicted by the information provided in the passage, which clearly
discusses both the positive and negative impacts of social media.

Section - Current Affairs, including General Knowledge


25 - c
In 1983, Nepalese Prime Minister Surya Bahadur Thapa was voted out of office
through a no-confidence motion. It was the first-ever no-confidence motion moved and
passed in the 20-year history of the Nepalese legislature presented by the informal
leader of the Opposition, Lokendra Bahadur Chand, who charged the Prime Minister of

70
seeking to undermine the active leadership of the Crown, not carrying out the directives
of King Birendra, for pursuing a wrong policy resulting in chaos etc.

26 - c
The general elections were held in Nepal from 18th February to 3rd April 1959 to elect
the 109 members of the first the House of Representatives and lower house of the
Parliament of Nepal. The elections were conducted under the provisions of the 1959
constitution adopted on 12th February. Hence, option (c) is correct.

27 - a
On March 8th, 2023, Ram Chandra Poudel, a veteran Nepali Congress leader and
former Deputy Prime Minister, was elected as the new President of Nepal. He secured a
comfortable victory with the vote of 214 lawmakers of Parliament and 352 provincial
assembly members. Poudel is the 3rd President of the country, succeeding Bidhya Devi
Bhandari, whose tenure ended on March 12th. Hence, option (a) is correct.

28 - d
The first king of Nepal was Prithvi Narayan Shah, a Gorkha ruler, who succeeded in
unifying the kingdoms of Kathmandu, Patan and Bhaktapur into a single state under the
Shah dynasty.

29 - c
On May 28, 2008, the newly elected Constituent Assembly declared Nepal a Federal
Democratic Republic, abolishing the 240 year-old monarchy. Nepal today has a
President as Head of State and a Prime Minister heading the Government. Currently,
Pushpa Kamal Dahal is the Prime Minister of Nepal.

30 - a
Comptroller and Auditor General of India (CAG) has been selected as external auditor
of the International Labour Organization (ILO), Geneva for a four-year term from 2024 to
2027, the apex auditor said in a statement. Girish Chandra Murmu is the CAG who will

71
take over from the incumbent external auditor of ILO, Supreme Audit Institution of
Philippines.

31 - b
In Geneva, a three-member team from CAG of India presented the strengths, approach
and skill set as also the vast experience of auditing international organisations to the
tripartite selection panel of ILO. It further said the selection panel was impressed by the
CAG's approach in developing a strategic partnership with ILO through which it aims to
assist ILO in meeting its strategic goals while maintaining critical independence and
oversight in performing the functions of the external auditor.

32 - c
International Labour Organisation
Explanation: The International Labour Organisation (ILO) has released its 11th edition of
the World of Work report, providing insights into the projected global unemployment
rates and the need for social protection. As the world recovers from the impact of the
pandemic, it is crucial to address the disparities in employment opportunities and foster
economic stability. Hence, option (c) is correct.

33 - c
World Food Day is observed on October 16 each year to emphasize the importance of a
healthy diet and a regular access to nourishing food. Hence, option (c) is correct.

34 - b
The World Bank is an international financial institution that provides loans and grants to
the governments of low and middle income countries for the purpose of pursuing capital
projects. It was established along with the International Monetary Fund at the 1944
Bretton Woods Conference. Hence, option (b) is correct.

72
35 - d
The Gujarat government signed a Memorandum of Understanding with computer
storage chip maker Micron for setting up a USD 2.75 billion semiconductor assembly
and test facility at Sanand in Ahmedabad district. Under the scheme, Micron will receive
50 per cent fiscal support for the total project cost from the Indian government and
incentives representing 20 per cent of the total project cost from the Gujarat
government.
Hence, option (d) is correct.

36 - c
Odisha Chief Minister Naveen Patnaik recently unveiled two groundbreaking initiatives
aimed at leveraging the power of Artificial Intelligence (AI) in the state. The initiatives,
named ‘Odisha for Artificial Intelligence’ and ‘Artificial Intelligence for Youth,’ are poised
to make a significant impact on digital literacy, technological advancements, and
economic growth. Hence, option (c) is correct.

37 - c
Chief Minister Bhupesh Baghel recently announced the launch of New Rural Housing
Scheme known as Gramin Awaas Nyay Yojana on 19th July 2023. This scheme aims at
providing free housing facility to the poor. Hence, option (c) is correct.

38 - b
The Pradhan Mantri Garib Kalyan Yojana (PMGKY) was launched in the year 2016 by
PM Narendra Modi along with the other provisions of the Taxation Laws (Second
Amendment) Act, 2016. It came into effect from 17th December 2016 under the Ministry
of Finance. Hence, option (b) is correct.

39 - c
Pilgrimage Rejuvenation and Spirituality Augmentation Drive (PRASAD) was launched
in the year 2015 under the Ministry of Tourism of India. Its aim is the integrated
development of pilgrimage destinations in a prioritised, planned, and sustainable

73
manner for providing a complete religious tourism experience. The focus of Pilgrimage
Rejuvenation And Spirituality Augmentation Drive- PRASAD is on the development and
beautification of the identified pilgrimage destinations. Hence, option (c) is correct.

40 - a
As has been said a million times, a year is an eternity in politics. Any one of a hundred
different things can upend the calculations of the smartest visualiser of the 2024
election scene. All, however, will agree that Karnataka has pumped life into speculation
about 2024. Moreover, speculation is leading to serious discussion. Those with proven
records of political success in different parts of India, people like Sharad Pawar,
Mamata Banerjee, Nitish Kumar and Arvind Kejriwal, seem to have concluded from the
Karnataka verdict that defeating Modi in 2024 may now be a distinct possibility. Hence,
option (a) is correct.

41 - b
“The strength of poor people,” said Rahul Gandhi, “defeated the power of crony
capitalists. This will happen in all states.” The message seemed to be that offering
shoulders and favourable policies to the unemployed, and to mothers, sisters and wives
groaning under rising prices, could become the winning theme that unites people
beyond caste and religion.
Hence, option (b) is correct.

42 - a
India celebrated its 13th National Voters Day on January 25. The first NVD was
celebrated in 2011. The Election Commission of India will host the celebrations every
year. ECI has released a logo marking the theme of the celebrations. On NVD, the
President of India will present the National Awards in the category of Electoral
Practices. Hence, option (a) is correct.

74
43 - a
The Tipra Motha Party was formed by the Tripuris, the indigenous tribe of the state.
They are demanding a separate state for their tribe. The state of Tripura follows a
unicameral legislature. There are 30 elected members in the assembly. The first Tripura
Legislative Assembly was formed in 1967. Hence, option (a) is correct.

44 - d
Manish Sisodia is the Deputy Chief Minister of Delhi holding the education portfolio. The
AAP party won the Punjab elections in 2022. According to the CBI, Manish was involved
in providing illegal support to the liquor companies. Eventually, he used the benefits to
win the Punjab elections in 2022. Hence, option (d) is correct.
45 - c
The Madras High Court was formed as a result of a merger of the Supreme Court of
Judicature at Madras, and Sadr Diwani Adalat. It has appellate jurisdiction over the
State of Tamil Nadu. The current Chief Justice of the High court is Sanjay V.
Gangapurwala.

46 - c
The head of Tamil Nadu’s panel on online gaming has called for a complete ban for
online gaming stating that regulation is not the solution. The state government is clear
that a ban is the only way out and it will have to be implemented, according to Justice K
Chandru, retired Madras High Court judge.

47 - b
The judges of the High Court are appointed by the President of India. The Chief Justice
is appointed by the President after consultation with the Chief Justice of India and the
concerned State Governor.

75
48 - b
The High Court is the highest court in a state in India. Articles 214 to 231 in the Indian
Constitution talk about the High Courts, their organisation, and powers. The Parliament
can also provide for the establishment of one High Court for two or more states.
For instance, Haryana, Punjab and the Union Territory of Chandigarh have a common
High Court. The northeastern states also have one common High Court. In addition,
Tamil Nadu shares a High Court with Puducherry.

49 - a
The first High Court in India is the Calcutta High Court. It was established in 1862 as the
High Court of Judicature at Fort William. Hence, option (a) is correct.
The head of Tamil Nadu’s panel on online gaming has called for a complete ban in this
space, arguing that regulation is not the solution. The state government is clear that a
ban is the only way out and it will have to be implemented, Justice K Chandru, retired
Madras High Court judge, told ET. A “legal battle” could be ahead between the state and
the Centre on this matter, said (2) who led the four-member committee tasked with
giving recommendations on online gaming in the state. Based on his panel’s report, an
ordinance was passed by the state assembly that sought to ban online gaming.

50 - b
Pope Francis, in collaboration with Santa Clara University's Markkula Center for Applied
Ethics, has released a comprehensive guidebook titled "Ethics in the Age of Disruptive
Technologies: An Operational Roadmap." This handbook addresses the ethical
challenges associated with emerging technologies, with a particular focus on artificial
intelligence (AI). The Pope's efforts aim to provide guidance to tech companies and
promote responsible AI development that aligns with human values and the common
good.

51 - c
Pope Francis, in collaboration with Santa Clara University's Markkula Center for Applied
Ethics, has released a comprehensive guidebook titled "Ethics in the Age of Disruptive

76
Technologies: An Operational Roadmap." Pope Francis's initiative reflects his belief in
the positive potential of technology while expressing deep concerns for the well-being of
humanity.

52 - a
UAE government launched an international conference on Artificial Intelligence (AI)
called the ‘Machines Can See 2023’ Summit on May 2023. The summit took place at
the Museum of the Future in Dubai and was organized in partnership between the
Artificial Intelligence, Digital Economy, and Remote Work Applications Office and
‘Machines Can See’ company.

53 - b
On 16 March, Microsoft unveiled its latest artificial intelligence (AI) integration with
Microsoft 365 Copilot. Designed to boost workplace productivity, the Copilot combines
the “power of large language models with business data and the Microsoft 365 apps to
unleash creativity, unlock productivity and uplevel skills,” according to a press
statement.

54 - a
John McCarthy is regarded as the father of Artificial Intelligence. The term was coined
by him and he was one of the founders of the discipline. He was one of the founding
fathers of artificial intelligence with Alan Turing, Marvin Minsky, Allen Newell and
Herbert A. Simon.

Section - Legal Reasoning

55 - b
Under torts, the claim is for unliquidated damages, whereas for breach of contract, the
suit is filed to seek liquidated damages which are already determined by the parties to
the contract while entering into the contract. In torts, the amount of compensation is not
predetermined by the parties to the suit, rather, it is at the discretion of the court to

77
assess the amount of harm or loss suffered by the plaintiff and grants the damages
accordingly.

56 - b
Reasonable foreseeability is a test of remoteness in the law of torts. Therefore,
according to this test, if the consequences of a wrongful act could have been foreseen
by a reasonable man, they are not too remote. A person is liable for the Doctrine of the
remoteness of damages in the law only when his wrongful conduct is directly related to
the effect of his action.

57 - a
The liability on the person who has committed a breach which may fairly and reasonably
be considered as arising naturally from the breach. It is assumed for the purpose that a
reasonable vendor understands the ordinary business practices and exigencies of his
buyer’s trade or business without the need for any special discussion or communication.
Therefore, Bachan Singh is liable to pay the compensation also along with damages for
the defective chain.

58 - d
When there is an absence of specific communication, a particular type of loss will fairly
and reasonably be regarded as having been in the contemplation of the parties for the
purpose of the general principle if there is a serious possibility or real danger or grave
risk or grave danger of its occurrence. A loss may seem to be recoverable where the
probability of its occurrence is less than an even chance but is nevertheless not very
unusual and easily foreseeable.
In the present case buyers being aware that the sellers could not obtain the goods
unless the letter of credit was provided, the sellers were entitled to loss of profit but not
any damages as it was a special loss, not within the contemplation of parties.

78
59 - b
When there is an absence of specific communication, a particular type of loss will fairly
and reasonably be regarded as having been in the contemplation of the parties for the
purpose of the general principle if there is a serious possibility or real danger or grave
risk or grave danger of its occurrence. A loss may seem to be recoverable where the
probability of its occurrence is less than an even chance but is nevertheless not very
unusual and easily foreseeable.
In the present case defendant will not be liable for loss of woman as this injury is not
foreseeable by defendant.

60 - c
In the case of R K Dalmia vs. Delhi Administration, the Supreme Court clarified that the
word 'property' is used in the Code in a much wider sense and not limited to 'moveable
property'. If not expressly given like in offence of theft the interpretation means that the
offence is applicable to both kinds of property.

61 - d
Section 405 of the Indian Penal Code states that the person who is alleged to have
committed this offence must possess a dishonest intention to convert or use or dispose
of the property that has been entrusted to him by the transferor. Furthermore, where the
alleged accused dishonestly disposes of the property and causes another person to
suffer from such disposal, then such a person is said to have committed the offence of
criminal breach of trust. Causing wrongful loss to the victim is not a pre-requisite for
offence of criminal breach of trust.

62 - d
In the case of State of UP v. Babu Ram Upadhya (2000), the apex Court held that if
there was an entrustment of property and the person taking dominion over such
property converted it to his own use. Therefore, an offence under Section 409 IPC was
held. In this case, the court decided on the similar facts.

79
63 - b
In Rashmi Kumar v. Mahesh Kumar Bhada (1996), the Supreme Court held that the wife
is the owner of Stridhan and that if she has entrusted her property to her husband. The
husband only has dominion over it. If he or any member of his family breaches such
entrustment and uses the same dishonestly for their own use or misappropriates it, they
shall be held liable for the offence of criminal breach of trust under Section 405 IPC.

64 - c
Section 405 incorporated under the Indian Penal Code defines elaborately the offence
of the criminal breach of trust. It states that in order to constitute the offence of criminal
breach of trust, it must be established that the accused was entrusted with the property
or with the dominion or power over the property of another person.

65 - a
The law of malicious prosecution states that there has to make its way between two
competing principles, -freedom of action that every man should have in bringing
criminals to justice and the necessity for checking lying accusations of innocent people.

66 - d
MUNISH only threatened but hasn’t initiated malicious prosecution therefore JAGDISH
cannot claim relief but he can get relief for intimidating and threatening him under other
laws.

67 - a
Plaintiff filed for relief against the defendant as the defendant initiated malicious
prosecution therefore the first requirement will be false prosecution by the defendant
against the plaintiff, not the other way round.

80
68 - d
Malicious prosecution is a common law tort, which found its origin in the ancient English
regime. The most important aspect of a person’s life is his life, liberty, and reputation.
The concept of malicious prosecution safeguards all three of them.

69 - b
Malicious prosecution is a common law tort, which found its origin in the ancient English
regime. The most important aspect of a person’s life is his life, liberty, and reputation.
The concept of malicious prosecution safeguards all three of them by effectively
protecting the person from unwanted and maliciously instituted proceedings. The origin
of this concept, in effect, came from the abuse of the judicial process.

70 - d
In the case of Carter v. White, (1883), a case with similar facts whereby the court held
that the Agent’s power to fill in the name of the drawer was not considered to be
terminated upon the death of the principal if the principal empowered the agent.

71 - a
In the case of Trueman v. Loder (1840), Court observed that it makes no sense whether
the agent intended to retain the contract on his own account in a specific situation. The
court stated that it was quite unfair to ask the principal to tell the whole world that he
had revoked his agent’s authority and that it should not be expected that he would
contact someone with whom the agent was willing to enter into a deal and inform him of
the termination.

72 - a
If, after the expiration of the time so stipulated in the contract, the parties continue their
relationship as principal and agent, a rebuttable presumption is raised that their
relations are governed by the original contract and that the contract is renewed for a
similar period. But, if the parties did not fix any appropriate time for the termination of
the contract, the contract is deemed to be terminated after a reasonable time.

81
73 - a
An agency may be revoked at the will of the principal when an agency is not coupled
with an interest, and no third party’s rights are involved. The party terminating the
agency must show good cause. The Principal cannot escape the obligation but he is
empowered to do whatever he desires to do with his business.

74 - c
An unpaid (gratuitous) agent will have no duty to act as no consideration has been
provided by the principal, but if he does act, he must do so in accordance with the
instructions set out by the principal.
In case, C has provided no consideration. But Consideration is not essential to create
an agency agreement. So, a valid agency is created.
Even though G is a gratuitous agent, he must do in accordance with the instructions set
out by the principal. So, C can take action against G for breach of his agency duties.

75 - d
Hindu Joint Family consists of all the family members. To cite all the male members
descended lineally up to any generation from a common ancestor together with their
mothers, wives, widows, and unmarried daughters. The status of daughters is
discussed in the case of Surjit Lal Chhabda v. CIT, Court held that a daughter remains a
part of her parents’ joint family, till the time she is unmarried. But, once she gets
married, she becomes a part of her husband’s Joint Hindu family.

76 - c
If the daughter is deserted by her husband or becomes a widow, and returns to her
father’s home permanently, she again becomes a part of her father’s Joint Hindu family.
But her children, remain part of their father’s Joint Hindu family.

82
77 - a
Without a common ancestor, a Joint Hindu Family cannot be brought into existence.
But, the presence of a common ancestor is a necessity only for its beginning, not for its
continuation i.e. the death of the common ancestor does not bring the Joint Hindu
Family to an end. Here, option (a) is the correct answer.

78 - b
On the death of the sole male member, a joint Hindu family can continue to exist at the
instance of already existing female members of the family. In CIT v. Sarwan Kumar, the
Supreme Court of India stated that on the death of the last surviving male member, the
other family members can continue to be a Joint Hindu family, even if they are not
coparceners.

79 - a
Hindu law has two main schools: the Mitakshara and dayabhaga school. The
Mitakshara school is a commentary on the code of Yajnavalkya and is written by
Vijnaneshwar. The Dayabhaga is a digest of all the codes and is written by
Jimutavahana.

80 - c
In the case of Padmavati v. Dugganaika, the driver of the jeep goes to fill petrol in it.
Two people ask for a lift. After some time, the jeep got toppled due to some issue with
the right wheel. The two strangers who took the lift were thrown out of the jeep and they
suffered some injuries leading to the death of one person.
Here, the Court held that the driver is not liable for any damages or offence because- It
was a case of an accident and the strangers had voluntarily gotten into the vehicle. But,
the principle of Volenti non fit injuria was not applicable here.

81 - b
If a person agrees to the publication of something about which he was aware of, then
such person cannot sue him on grounds of defamation.

83
82 - b
In Lakshmi Rajan v. Malar Hospital, a married woman aged forty noticed a lump in one
of her breasts but this pain does not affect her uterus. After the operation, she saw that
her uterus has been removed without any justification. The hospital authorities were
made liable for the removal of her uterus as her consent was taken for the treatment of
the lump and not for removing the uterus.

83 - a
In the case of Hegarty v. Shine, Court observed that mere concealment of facts is not
considered to be a fraud to vitiate consent. Hence, the action will fail because mere lack
of disclosure of facts does not amount to fraud based on the principle ex turpi causa
non oritur actio i.e. no action arises from an immoral cause.

84 - a
In the case of Smith v. Baker, the plaintiff worked on a drill to cut rocks. Some stones
were being conveyed using a crane over his head. Once a stone fell on his skull
causing some injuries. The court held that the defendants were negligent as they did not
inform him before. Mere knowledge of risk does not mean that the person has
consented to risk. The maxim volenti non fit injuria has no application. But, if a person
ignores the instructions given by the employer thereby suffering injury then this maxim
applies.

Section - Logical Reasoning

85 - b
The passage discusses how utility companies with monopolies in selling electricity and
gas are using their political power to slow down the clean energy transition. They
manipulate the pricing by including their political costs in rates alongside their
investments, which makes it difficult for customers to avoid supporting their agenda.

84
This indicates that energy markets are vulnerable to manipulative pricing practices by
these utility companies.
Reasons for incorrect options:
a) Reduction Act has to be redesigned to offer more to utility companies.
This option is not supported by the passage. The Inflation Reduction Act is mentioned in
the context of providing tax incentives to utilities for increasing their investments in clean
energy, but there is no indication that it needs to be redesigned to offer more to utility
companies.
c) The two big transitions are enough to reverse climate change.
The passage discusses the two big transitions required to combat climate change,
which are generating electricity from clean sources and retooling everything else to run
on clean electricity. However, it does not claim that these two transitions alone are
enough to reverse climate change. The passage focuses more on the obstacles and
challenges posed by utility companies.
d) Many customers have switched to homemade electricity options.
The passage does not mention anything about customers switching to homemade
electricity options. It discusses how utility companies view distributed energy, such as
rooftop solar owned by customers, as a threat to their business and have tried to hinder
its adoption. However, there is no information suggesting that many customers have
already switched to homemade electricity options.

86 - a
a) Utility companies fund the election campaign of candidates that oppose cleaner
energy.
This option would provide strength to the argument of the author against utility
companies. If it is true that utility companies are actively funding the election campaigns
of candidates who oppose cleaner energy and are likely to support their agenda, it
would further demonstrate the extent of their political power and their attempts to
influence policies in favor of dirty energy sources. This would support the author's
argument that utility companies are using their political power to slow down the clean
energy transition.

85
b) Government lacks the willpower to deviate from coal-based electricity.
This option is not directly related to the argument against utility companies. While it may
be a valid concern for the transition to cleaner energy sources, it does not provide
strength to the author's argument about utility companies using manipulative strategies
to hinder the clean energy transition.
c) No research is going on to make renewable energy cheap.
This option is not directly related to the argument against utility companies either. While
research on making renewable energy cheaper would be beneficial for promoting clean
energy adoption, the passage primarily focuses on the role of utility companies in
hindering the transition and their manipulative practices.
d) Customers do not understand the negative manipulative strategy of utility companies.
This option could potentially provide some support to the argument against utility
companies. If customers are unaware of the manipulative strategies used by utility
companies, it may explain why the companies can continue their practices without
significant opposition. However, it is not as strong as option (a), which directly points to
the political power and influence of utility companies.

87 - b
This option would weaken the claims of the author against utility companies. If it is true
that most of the candidates supported by utility companies, who oppose cleaner energy,
have lost recent elections, it suggests that the utility companies' political influence may
not be as significant as claimed by the author. This weakens the argument that utility
companies are using their political power to effectively hinder the clean energy
transition.
Incorrect options-
a) Utility companies spend more money on research on clean energy than negative
campaigns.
This option is not directly related to the author's claims against utility companies. It may
show that utility companies are investing in clean energy research, but it doesn't
address the issue of their manipulative strategies or political power.

86
c) Customers are not willing to deviate from cheap dirty energy to expensive cleaner
energy.
This option doesn't directly weaken the claims of the author against utility companies. It
highlights a potential challenge in transitioning to cleaner energy sources, but it doesn't
provide any evidence to counter the argument that utility companies may be using
manipulative tactics to slow down the clean energy transition.
d) The technology to store renewable energy is not effective enough.
This option is not directly related to the claims against utility companies either. It
discusses a limitation in renewable energy technology, but it doesn't address the
author's argument about utility companies using their political power to hinder the clean
energy transition.

88 - b
The paradox present in the argument of the author is option (b). The author argues that
utility companies are using their political power to slow down the clean energy transition,
essentially blocking the growth of clean energy. However, the paradox arises when it is
mentioned that utility companies are also spending on research on clean energy. This
contradiction implies that while they may invest in research, their actions and strategies
are still hindering the progress of clean energy adoption, raising questions about their
true intentions and commitment to promoting cleaner sources of energy.

a) Government projects schemes like the ‘Reduction Act’ but officials are corrupt.
This option describes a situation where the government introduces beneficial schemes
like the 'Reduction Act,' but the officials implementing these schemes are corrupt,
leading to a paradox. However, this paradox is not present in the argument of the author
against utility companies. The author's argument focuses on utility companies using
their political power to slow down the clean energy transition, and there is no mention of
corrupt government officials in the context of the author's argument.

c) Customers continue to buy dirty energy understanding the implications of it.

87
This option describes a situation where customers knowingly continue to buy dirty
energy despite understanding its implications, creating a paradox. While it may be a
challenging aspect in promoting clean energy adoption, it is not directly related to the
argument against utility companies presented by the author. The author primarily
discusses the role of utility companies in hindering the clean energy transition, not the
behavior of customers.
d) Energy companies are blamed not the government for letting them play dirty.
This option describes a situation where energy companies are blamed for their
practices, but the government is not held accountable for allowing such behavior,
creating a paradox. While this may be a relevant concern in some cases, it is not a
paradox present in the argument of the author. The author's focus is on utility
companies' actions and their political power, not on blaming the government for the
companies' behavior.

89 - a
Option a) "He has not supported his claims with the data and facts" is indeed the
correct fault found in the author's argument. The passage does not provide extensive
data and facts to support the claims made against utility companies. While it mentions
some general aspects of utility companies' actions, the argument could have been
strengthened further by providing more concrete evidence and specific examples.
Options b) "He undermines the fact retooling everything is not feasible," c) "He does not
appreciate the fact that cost is the major factor for customers," and d) "He has not
provided the rationale for the approach of the utility companies" are not directly
supported by the information provided in the passage. The author's focus in the
passage is primarily on the role of utility companies in slowing down the clean energy
transition, not on undermining the feasibility of retooling, dismissing customer cost
concerns, or explaining the rationale of utility companies' approach.

90 - c
c) A restaurant promotes plant-based options but increases prices for these dishes.

88
The correct option (c) parallels the situation described in the passage about utility
companies hindering the clean energy transition. In the passage, it is mentioned that
utility companies invest in clean energy but simultaneously hinder the growth of clean
energy adoption through political power. Similarly, in option (c), the restaurant promotes
plant-based options (which are considered environmentally friendly) but increases
prices for these dishes, making them less affordable for customers. Both scenarios
involve entities outwardly endorsing environmentally friendly initiatives but engaging in
actions that contradict their purported commitment.
Reasons for incorrect options:
a) A pharmaceutical company funds research for a cure but lobbies against affordable
access to existing treatments.
This option does not parallel the situation in the passage. The passage discusses utility
companies investing in clean energy research while hindering the adoption of clean
energy. Option (a) involves a pharmaceutical company funding research for a cure but
lobbying against affordable access to existing treatments, which is a different scenario
and not directly related to the utility companies' situation in the passage.
b) A tech company develops innovative gadgets but offers discounts on older, less
eco-friendly models.
This option also does not parallel the situation in the passage. The passage discusses
utility companies investing in clean energy research while hindering the growth of clean
energy adoption. Option (b) involves a tech company developing innovative gadgets
while offering discounts on older, less eco-friendly models, which is a different context
and not directly related to the utility companies' situation in the passage.
d) A fashion retailer introduces a sustainable clothing line but follows harmful production
practices.
This option does not parallel the situation in the passage. The passage discusses utility
companies investing in clean energy research while hindering the clean energy
transition. Option (d) involves a fashion retailer introducing a sustainable clothing line
but following harmful production practices, which is a different context and not directly
related to the utility companies' situation in the passage.

89
91 - b
This statement weakens the argument that violence is the sole reason the world is not
on track to end hunger by 2030. The passage primarily attributes hunger and food
insecurity to violence and conflicts in various regions. However, if it is true that the
agricultural output has reduced over time, it suggests that other factors, such as
changes in agricultural practices, climate-related issues, or resource constraints, could
also be contributing to food insecurity, not solely violence and conflict. This weakens the
claim that violence is the only reason for the world not being on track to end hunger by
2030.
Reasons for incorrect options:
a) The number of violence-hit countries is very less.
This statement is not relevant to weakening the argument about violence being the sole
reason for the world not being on track to end hunger. The argument is not about the
number of violence-hit countries, but rather about the impact of violence and conflict on
food insecurity.
c) There is enough food to sustain the current population.
This statement also does not weaken the argument about violence being the sole
reason for not ending hunger by 2030. While it suggests that there might be enough
food to sustain the current population, the argument focuses on how violence and
conflict disrupt food systems and contribute to food insecurity, regardless of the total
food availability.
d) Food production is highly concentrated only in a few countries.
This statement is not directly relevant to weakening the argument about violence being
the sole reason for not ending hunger by 2030. It talks about the concentration of food
production in specific countries but does not address the impact of violence and conflict
on food security, which is the primary concern in the passage.

92 - c
The passage discusses the relationship between violence and conflict with food
insecurity. It highlights that countries experiencing acute food insecurity, such as
Afghanistan, Burkina Faso, Ethiopia, Mali, Sudan, and Syria, are also facing deadly

90
conflicts. This suggests that vulnerable populations, living in conflict-affected regions,
are more significantly impacted by hunger and food insecurity. The passage implies that
violence and conflict disproportionately affect these vulnerable populations, leading to
acute food insecurity in these regions.
Reasons for incorrect options:
a) The upcoming meeting of heads of government will focus on reducing violence.
The passage does not provide information about the specific agenda or focus of the
upcoming meeting of heads of government. While it mentions the meeting, it does not
suggest that the focus will be solely on reducing violence.

b) International economic sanctions effectively address food insecurity in


conflict-affected countries.
The passage mentions that coercive measures like international economic sanctions
against warring countries can contribute to hunger by disrupting food systems.
However, it does not imply that these sanctions effectively address food insecurity in
conflict-affected countries. It highlights their negative impact on food systems instead.
d) The report's findings will lead to immediate policy changes.
The passage does not explicitly state that the report's findings will lead to immediate
policy changes. It discusses the alarming situation of hunger and conflict, and the need
for actions to address it, but does not provide information on the immediate impact of
the report's findings on policy changes.

93 - b
b) War leads to disruption in food supply but sanctions on warring nations are not
recommended either.The passage highlights the paradox that war and violence lead to
disruptions in food supply, contributing to food insecurity in conflict-affected regions.
However, the passage also mentions that international economic sanctions, which are a
coercive measure against warring countries, can also disrupt food systems and
contribute to hunger. This paradoxical situation suggests that both the presence and
absence of certain actions (war and sanctions) can lead to negative impacts on food

91
security, leaving the reader with the question of how to address food insecurity in
conflict-affected regions effectively.
Food Insecurity is not related to food production and distribution but to violence.
Option (a) is not supported by the passage. The passage actually emphasizes the
relationship between violence and food insecurity, suggesting that food insecurity is
related to violence.
c) War leads to disruption in food supply but sanctions on warring nations are not
recommended either.
Option (c) is not mentioned in the passage. There is no information provided about the
economic status of the countries with food insecurities mentioned in the passage.

94 - b
The passage mentions that parties to a conflict may weaponize food, suggesting that it
is used as a strategic tool to exert power, gain leverage, or influence outcomes in a
conflict. This implies that manipulating or controlling the production, distribution, or
access to food can be employed as a tactic in conflicts.
Option a: Manipulating or controlling the production, distribution, or access to food.
While manipulating or controlling the production, distribution, or access to food is related
to the purpose of 'weaponizing food,' it does not encompass the full context of exerting
power, gaining leverage, or influencing outcomes in a conflict.
Option c: Using food as a strategic tool to arm-twist international organizations like
United Nations.
The passage does not mention using food as a strategic tool to arm-twist international
organizations like the United Nations.
Option d: Establishing pressure on the warring countries to inflict sanctions.
The passage does not discuss establishing pressure on warring countries to inflict
sanctions for the purpose of 'weaponizing food.'

95 - d
The passage does not provide explicit information to conclude any specific statement
about the Sustainable Development Goals (SDGs). However, based on the passage, it

92
can be inferred that all of the statements could potentially apply to the SDGs. The
passage highlights that the world is not on track to end hunger and malnutrition by
2030, a promise made as part of the SDGs. It also mentions that crucial efforts to study
and implement policies to end hunger are hampered by violence. These aspects
suggest that there may be a lack of due consideration, lack of effort, and challenges in
achieving the SDGs. The inferences made are based on the challenges mentioned in
the passage but do not conclusively support any one specific statement.

96 - b
The presence of violence and conflict significantly hinders progress towards achieving
the SDG to end hunger and malnutrition by 2030.
The passage suggests that violence and conflict are in fact the primary causes of
hunger worldwide and that they are pivotal reasons for the world not being on track to
achieve the SDG to end hunger and malnutrition by 2030. The passage highlights the
connection between violence and food insecurity, mentioning how conflicts endanger
food security when crops are destroyed, food supplies are disrupted, and coercive
measures like international economic sanctions contribute to hunger. Therefore, it can
be inferred that the presence of violence and conflict significantly hinders progress
towards achieving the SDG to end hunger and malnutrition by 2030.
Reasons for incorrect options:
a) Achieving the SDG to end hunger and malnutrition by 2030 is possible without
addressing violence and conflict.
This option is not supported by the passage. The passage clearly states that violence
and conflict are the primary causes of hunger worldwide and are pivotal reasons for the
world not being on track to achieve the SDG. It implies that addressing violence and
conflict is essential to making progress towards the SDG, making option (a) incorrect.

c) The SDG to end hunger and malnutrition by 2030 does not prioritize addressing the
impact of violence and conflict on food insecurity.
The passage does not provide information about the prioritization of addressing the
impact of violence and conflict in the SDG to end hunger and malnutrition. It focuses on

93
the connection between violence and food insecurity, but it does not discuss the SDG's
specific priorities. Therefore, option (c) cannot be inferred from the passage.
d) The relationship between violence and achieving the SDG to end hunger and
malnutrition by 2030 is not clearly established in the passage.
This statement is not accurate. The passage clearly establishes the relationship
between violence and the hindrance to achieving the SDG to end hunger and
malnutrition by 2030. It states that violence and conflict are the primary causes of
hunger worldwide and are pivotal reasons for the world not being on track to achieve
the SDG. Therefore, option (d) is incorrect.

97 - c
The main purpose of the author in the passage is to provide readers with historical
information and context about patents. The passage traces the origin of patents and
their historical development, starting from as far back as 500 B.C. in a city dominated by
gourmands to the systematic use of monopoly privileges for inventors in Venice in the
15th Century and the awards given to inventors by German princes in the 16th Century.
The author is not primarily focusing on the benefits of patents or the needs of patents,
but rather on presenting a historical account of the evolution of patents and their use as
a stimulus for encouraging industrial innovation. Option (c) captures this intention of the
author, making it the correct answer.
Reasons for incorrect options:
a) He is looking to highlight the fact that patents can lead to a monopoly.
This option is not supported by the passage. While the passage does mention that
patents grant a monopoly to inventors, it does not focus on highlighting the potential
monopolistic aspect of patents. Instead, it presents a broader historical perspective on
patents.
b) The author is describing the benefits of patents to avoid monopolistic markets.
The passage does not primarily focus on describing the benefits of patents to avoid
monopolistic markets. While it mentions that patents grant a monopoly to inventors, it
does not delve into the discussion of how patents can prevent monopolistic markets or
their role in doing so.

94
d) It is to give readers an insight into the needs of patents.
The passage does not specifically provide insights into the needs of patents. While it
mentions the purpose and underlying economic justification for the patent system as a
stimulus for investment in industrial innovation, it does not elaborate extensively on the
needs or requirements of patents.

98 - d
d)Based on the facts given in the passage, it can be inferred that the concept and
design of patents have evolved over time, but the objective behind patents remains
consistent. The passage traces the historical development of patents, starting from as
far back as 500 B.C. in a city dominated by gourmands to the systematic use of
monopoly privileges for inventors in Venice in the 15th Century and the awards given to
inventors by German princes in the 16th Century. These historical instances show that
the concept of patents has been present in different eras, and the passage highlights
how the grant of exclusive rights to inventors has been a common practice across
different periods.
Option (a) People of different eras find patents useful to an extent only.
Option (a) is not directly supported by the passage. While the passage mentions
different historical eras, it does not explicitly state that people of those eras found
patents useful only to an extent.
Option (b) Different eras considered monopoly privileges as patents.
Option (b) is not entirely accurate. The passage does mention that the grants to
inventors in the past can be considered as monopoly privileges, but it does not imply
that different eras exclusively equated monopoly privileges with patents. The passage
provides broader information about the historical context of patents.
Option (c )- The concept of patent is a generation of business society.
Option (c) is not supported by the passage. The passage does not mention that the
concept of a patent is exclusively a generation of the business society.
Option (d) is the correct inference because it captures the idea that the design of
patents has changed over different eras, but the objective of granting exclusive rights to

95
inventors to encourage innovation has remained consistent throughout history, as seen
in various examples presented in the passage.

99 - b
Based on the information provided in the passage, it can be logically assumed that
knowledge of the rules of patents can lead to encouraging creativity. The passage
mentions that patents are a form of protection for the creative work of the human mind
and act as a stimulus to investment in industrial innovation. Patents grant inventors
exclusive rights to control the output and, within the limits set by demand, the price of
the patented products. This protection of creative work through patents can incentivize
inventors to invest in innovation and pursue new ideas, knowing that they will have the
legal protection and exclusive rights to their inventions.
Option (a) - Patents imbibe creativity in artists and inventors.
Option (a) cannot be assumed from the passage as it does not directly mention patents
imbuing creativity in artists and inventors. The passage discusses patents as a form of
protection for creative work, but it does not explicitly imply that patents themselves
imbibe creativity in artists and inventors.
Option (c ) -Creativity and inventions can be saved only by patenting.
Option (c) is not supported by the passage. The passage does not state that creativity
and inventions can be saved only by patenting. While patents can protect inventions
and provide incentives for innovation, there are other forms of protection and
encouragement for creativity as well.
Option (d) - Patents lead to a monopoly that kills inventions in the long term.
Option (d) is not supported by the passage. The passage mentions that patents grant a
monopoly to inventors, but it does not suggest that patents lead to a monopoly that kills
inventions in the long term. Instead, it focuses on the positive aspects of patents as a
stimulus for investment in innovation.
Therefore, option (b) is the best logical assumption based on the information provided in
the passage.

96
100 - a
Option (a) strengthens the argument propagated by the author in favor of patents. The
passage discusses patents as a form of protection for creative work and highlights that
patents grant the inventor exclusive rights to control the output and price of the patented
products. By mentioning that patents can provide an important source of revenue for a
business, this option supports the idea that patents offer economic benefits and
incentives for inventors and businesses. It aligns with the underlying economic and
commercial justification for the patent system mentioned in the passage, which is to act
as a stimulus to investment in industrial innovation.
Reasons for incorrect options:
b) Patent means making certain technical information about your invention publicly
available.
This option is not directly related to strengthening the argument in favor of patents.
While patents do require the disclosure of technical information about the invention, the
passage does not focus on this aspect to promote the benefits of patents or their role in
encouraging creativity and innovation.
c) Taking action against an infringer can be a very expensive process.
This option does not directly strengthen the argument in favor of patents. While it does
highlight one aspect of the patent system related to enforcing patent rights, it does not
provide specific support for the broader argument about the benefits and incentives of
patents for creativity and industrial innovation.
d) You can believe your invention has potential in other countries’ markets as well.
This option is not directly related to strengthening the argument in favor of patents.
While it touches on the potential international scope of inventions, it does not provide
specific support for the argument presented in the passage about the role of patents in
encouraging creativity and innovation.

101 - b
Option (b) An invention whose primary goal or intended use is in accordance with the
law. According to the passage, patents are a form of protection for the creative work of
the human mind, and the main motivation for such protection is to encourage creative

97
activity. The passage specifically mentions that patents, like the grant of exclusive rights
to inventors, enable the inventor to control the output and, within the limits set by
demand, the price of the patented products. This underlying economic and commercial
justification for the patent system is that it acts as a stimulus to investment in industrial
innovation.
Option (b) states that the invention's primary goal or intended use is in accordance with
the law. Inventions that align with the law and have legitimate, non-trivial applications
are suitable for patents, as they can be considered genuine and meaningful innovations
that contribute to industrial development. Therefore, option (b) is consistent with the
purpose and justification for patents as presented in the passage.
Reasons for incorrect options:
a) Discovery of a scientific principle
Patenting scientific principles or discoveries is typically not eligible for patents. Patents
are granted for inventions that are novel, non-obvious, and have a practical, tangible
application. Discoveries of scientific principles, being fundamental knowledge, are
usually considered non-patentable subject matter.
c) Any invention that might be frivolous or trivial.
Frivolous or trivial inventions are generally not eligible for patents. Patents are granted
for significant and non-obvious inventions that have practical utility and contribute to
industrial development.
d) An invention which, in effect, is traditional knowledge.
Inventions that are based on traditional knowledge or are considered part of the public
domain may not be eligible for patents. Patents are granted for novel and non-obvious
inventions, and if an invention is already known or part of the traditional knowledge, it
may not meet the criteria for patentability.

102 - a
The argument presented in the passage is that the patent system acts as a stimulus to
investment in industrial innovation. The passage mentions that patents are a form of
protection for the creative work of the human mind and enable the inventor to control
the output and price of the patented products. By granting exclusive rights to inventors,

98
the patent system incentivizes them to invest in industrial innovation, knowing that they
will have legal protection and control over their inventions. This assumption is crucial to
the argument because it supports the economic and commercial justification for the
patent system as a means of encouraging creativity and investment in innovation.
Reasons for incorrect options:
b) All forms of creative work are eligible for patent protection.
This option is not supported by the passage. The passage discusses patents as a form
of protection for the creative work of the human mind but does not imply that all forms of
creative work are eligible for patent protection. In fact, patents are granted for specific
types of inventions that meet the criteria of novelty, non-obviousness, and practical
utility.
c) Patents are primarily granted to protect the interests of large corporations and
businesses.
This option is not supported by the passage. While the passage highlights the economic
and commercial justification for the patent system, it does not specifically mention that
patents are primarily granted to protect the interests of large corporations and
businesses. Patents are granted to individual inventors and entities that meet the
requirements for patentability, regardless of their size.
d) The patent system is the only means of protecting intellectual property and
encouraging creativity.
This option is not supported by the passage. While the passage discusses the role of
patents in protecting intellectual property and encouraging creativity, it does not claim
that the patent system is the only means of doing so. There are other forms of
intellectual property protection, such as copyrights and trademarks, that also contribute
to the protection of creative work and innovatio

103 - c
The author presents an argument against the "marginal productivity" theory in
microeconomics, stating that this theory has been shown to be logically contradictory as
a result of the "capital controversy." The passage mentions that mainstream
microeconomics has attempted to provide a theory of profit (or what it calls interest)

99
through the "marginal productivity" theory, but it has been discredited due to logical
contradictions. The "capital controversy" refers to a theoretical debate among
economists regarding the validity of the "marginal productivity" theory of profit.
Reasons for incorrect options:
a) The theory contradicts the concept of profit.
This option is not explicitly mentioned in the passage. While the passage mentions that
the "marginal productivity" theory has been shown to be logically contradictory, it does
not specifically state that the theory contradicts the concept of profit. The focus of the
passage is on contrasting Marx's theory of profit with mainstream economic theories.

b) The theory lacks explanatory power.


While the passage mentions that mainstream economic theories, including the
"marginal productivity" theory, provide almost no theory of profit and have little or no
explanatory power compared to Marx's theory, the specific argument against the
"marginal productivity" theory is its logical contradiction, not just its lack of explanatory
power.
d) The theory is being dropped from microeconomics textbooks.
This option is mentioned in the passage but is not the main argument presented against
the "marginal productivity" theory. The passage does mention that the theory is being
quietly dropped from microeconomics textbooks at both the graduate and
undergraduate levels, but it is not the primary argument against the theory. The main
argument presented is that the theory has been discredited due to logical
contradictions.

104 - d
If true, option (d) would weaken the author's argument that Marx's theory of profit is
superior to mainstream economic theories. The author presents Marx's theory as
logically robust and having very impressive explanatory power. However, if leading
economists from various schools of thought criticize Marx's theory for its lack of
empirical evidence, it raises doubts about the validity and reliability of the theory.
Empirical evidence plays a crucial role in supporting and validating economic theories,

100
and criticism from prominent economists regarding the lack of empirical evidence
weakens the credibility of Marx's theory.
Reasons for incorrect options:
a) Mainstream microeconomics has recently developed a new theory of profit that
addresses the flaws in the "marginal productivity" theory.
This option does not directly weaken the author's argument in favor of Marx's theory.
The passage states that the "marginal productivity" theory has been discredited due to
logical contradictions, and the presence of a new theory addressing its flaws does not
necessarily negate the superiority of Marx's theory as presented by the author.
b) Marx's theory of profit fails to account for the role of technological advancements in
capitalist production.
This option is not mentioned in the passage, and its validity is not supported by the
information provided. The passage focuses on the economic and commercial
justification for the patent system and the comparison between Marx's theory and
mainstream economic theories, but it does not discuss the role of technological
advancements in capitalist production.
c) Mainstream macroeconomic theories have been successful in predicting economic
trends and outcomes.
This option does not directly weaken the author's argument about Marx's theory of
profit. The passage briefly mentions that mainstream macroeconomic theory has no
theory of profit at all, but it does not delve into the success or failure of mainstream
macroeconomic theories in predicting economic trends and outcomes. The focus of the
passage is on the lack of a theory of profit in mainstream macroeconomics.

105 - a
Option (a) provides the most logical way for mainstream theories to return to the
textbooks. The passage suggests that mainstream economic theories, including the
"marginal productivity" theory of profit, have been discredited due to logical
contradictions and lack of explanatory power. For these theories to regain credibility and
be included in textbooks, it would be essential to address and resolve the flaws
identified through rigorous academic research and scholarly debates. This process of

101
critical examination and refinement is a fundamental part of academic progress and can
lead to the improvement and reintegration of mainstream theories into economics
textbooks.
Reasons for incorrect options:
b) Mainstream economics textbooks include a separate section discussing alternative
theories, including Marx's theory of profit.
This option suggests including alternative theories, including Marx's theory of profit, in a
separate section. While this may be a way to present diverse perspectives, it does not
directly address the issue of regaining mainstream theories' credibility and inclusion in
textbooks.
c) Leading economists advocate for the inclusion of mainstream theories in textbooks to
provide a comprehensive understanding of economic principles.
While including mainstream theories in textbooks might be advocated by leading
economists to provide a comprehensive understanding of economic principles, it does
not specify how the flaws in these theories would be addressed or whether they would
regain credibility.
d) Mainstream economics departments collaborate with experts in Marxian economics
to integrate both perspectives into their curricula.
This option suggests collaboration between mainstream economics departments and
experts in Marxian economics to integrate both perspectives into their curricula. While
this could foster a more comprehensive approach to economics, it does not directly
address how the flaws in mainstream theories would be resolved and how they would
return to economics textbooks.

106 - a
The passage implies that workers are exploited under capitalism, as Marx's theory of
profit suggests that capitalists generate profits through the surplus labour of workers.
This suggests that workers may receive lower wages than the value of their labour,
leading to income inequality and struggles to make ends meet. Therefore, option a,
which states that many workers in capitalist economies face income inequality and

102
struggle to make ends meet, aligns with the inference based on the information
provided in the passage.
Option b: The passage does not specifically mention that workers in capitalist societies
are rewarded based on their skills and qualifications. It focuses on the exploitation of
workers rather than the direct relationship between skills and rewards.
Option c: The passage does not explicitly state that exploitation is prevalent in all
capitalist countries, regardless of their economic development. It emphasizes the
exploitative nature of capitalism in general.
Option d: The passage does not mention the necessity of comprehensive social welfare
programs for labourers in capitalist economies. It focuses on the exploitation of workers
rather than the provision of social welfare programs.

107 - b
Option (b) follows from the passage about socialism. The passage presents Marx's
theory of profit, which concludes that capitalism is inherently and unavoidably an unjust
and exploitative economic system. It suggests that profit in capitalism is the result of the
exploitation of workers because the value produced by workers is greater than the
wages they are paid. In contrast, the passage suggests that if a just and equitable
economic system without exploitation is desired, then Marx's theory suggests that the
economic system should be changed from capitalism to socialism. Therefore, it can be
assumed that the adoption of socialism has the potential to minimize exploitation, as it is
presented as an alternative to capitalism, which is deemed inherently exploitative in the
passage.
Reasons for incorrect options:
a) Socialism is a utopian economic system with no grievances of labours.
This option is not supported by the passage. The passage discusses Marx's theory of
profit and suggests that capitalism is exploitative. It mentions socialism as an alternative
to capitalism to address the exploitation of workers, but it does not portray socialism as
a utopian economic system with no grievances of labor.

c) Unlike capitalism, socialism ensures profits and social equality as well.

103
This option is not supported by the passage. The passage focuses on the exploitation of
workers under capitalism and presents socialism as an alternative. While it suggests
that the profit of capitalists is the result of the exploitation of workers in capitalism, it
does not explicitly state that socialism ensures profits and social equality.

d) Socialism might have its drawbacks, but it is better than capitalism.


This option is not directly supported by the passage. While the passage presents
socialism as an alternative to capitalism to address exploitation, it does not explicitly
compare the drawbacks of both systems or state that socialism is better than capitalism.
It primarily focuses on the economic and commercial justification for the patent system
and the comparison between Marx's theory and mainstream economic theories.

108 - a
Option (a) Patents provide legal protection for creative work, leading to increased
investment in industrial innovation.
The correct option is (a) because it can be inferred from the information provided in the
passage. The passage discusses the economic and commercial justification for the
patent system and highlights that patents are a form of protection for the creative work
of the human mind. It mentions that patents grant inventors exclusive rights to control
the output and price of the patented products. This protection of creative work through
patents acts as a stimulus to investment in industrial innovation. When inventors and
innovators know that their ideas and inventions will be legally protected through patents,
it encourages them to invest in industrial development and pursue new ideas, as they
will have the exclusive rights to their inventions, enabling them to control their output
and profits.
Reasons for incorrect options:
b) The patent system is the sole means of encouraging creativity and investment in
industrial development.
This option is not supported by the passage. While the passage highlights the role of
patents in encouraging innovation and investment in industrial development, it does not

104
state that the patent system is the sole means of doing so. There may be other forms of
protection and incentives for innovation that complement the patent system.
c) Mainstream economic theories acknowledge the superior role of patents in
stimulating innovation.
This option is not supported by the passage. The passage contrasts mainstream
economic theories with Marx's theory of profit, but it does not specifically mention
whether mainstream economic theories acknowledge the superior role of patents in
stimulating innovation. It focuses on the lack of a theory of profit in mainstream
macroeconomics and the flaws in the "marginal productivity" theory in mainstream
microeconomics.
d) Marx's theory of profit offers a more robust explanation for the role of patents in
industrial development.
This option is not directly supported by the passage. While the passage presents Marx's
theory of profit and highlights its role in addressing exploitation, it does not compare
Marx's theory with mainstream economic theories regarding the role of patents in
industrial development. The focus of the passage is on patents as a form of protection
for creative work and their economic and commercial justification.

Section - Quantitative Techniques

109 - b

To find the number of employees surveyed with a bachelor's degree, we need to calculate the
percentage of employees with a bachelor's degree and apply it to the total number of employees
surveyed.

We are given the following information:

- Total number of employees surveyed: 800

- Percentage of employees with a bachelor's degree: 30%

105
Number of employees with a bachelor's degree = Percentage of employees with a bachelor's
degree ×* Total number of employees surveyed

= 30% ×* 800

To calculate this, we can convert the percentage to a decimal:

30% = 30/100 = 0.30

Number of employees with a bachelor's degree = 0.30 ×* 800

= 240

Therefore, the correct answer is option (b) 240.

110 - b

To find the percentage of employees surveyed with at least a master's degree, we need to
calculate the cumulative percentage of employees with a master's degree and a Ph.D.

We are given the following information:

- Total number of employees surveyed: 800

- Percentage of employees with a master's degree: 20%

- Percentage of employees with a Ph.D.: 10%

Percentage of employees with at least a master's degree = Percentage of employees with a


master's degree + Percentage of employees with a Ph.D.

= 20% + 10%

= 30%

106
Therefore, the correct answer is option (b) 30%.

111 - c

If the total number of employees surveyed is 1200 instead of 800, we can calculate the number of
employees with a high school diploma by scaling up the given percentage.

According to the original survey, 40% of the employees had a high school diploma. To find the
number of employees with a high school diploma in the larger survey, we can use the ratio of the
total number of employees surveyed.

Let's calculate it:

Original survey:

Total employees surveyed: 800

Number of employees with a high school diploma: 40% of 800 = (40/100) ×* 800 = 320

Larger survey:

Total employees surveyed: 1200

Number of employees with a high school diploma: (40/100) ×* 1200 = 480

Ratio should be = original value : updated value of survey

Ratio should be = 320 : 480

Ratio should be = 2 : 3

Therefore, in the larger survey of 1200 employees, there would be 480 employees with a high
school diploma , now the ratio should be 2 : 3

112 - c

To determine the minimum percentage of women among all the employees surveyed, we need to
consider the percentage of women in each educational category and calculate the overall
minimum percentage.

107
According to the given information:

- Percentage of employees with a high school diploma: 40%

- Percentage of employees with a bachelor's degree: 30%

- Percentage of employees with a master's degree: 20%

- Percentage of employees with a Ph.D.: 10%

We are also provided with the following additional information:

- 40% of employees with a high school diploma are women.

- 20% of employees with a Ph.D. are women.

To calculate the minimum percentage of women among all the employees surveyed, we need to
find the minimum contribution from each educational category and sum them up.

Percentage contribution from employees with a high school diploma:

Women with a high school diploma = 40% of 40% (women among employees with a high school
diploma)

= 0.4 ×* 40% = 0.4 ×* 0.4 = 0.16 or 16%

Percentage contribution from employees with a Ph.D.:

Women with a Ph.D. = 20% of 10% (women among employees with a Ph.D.)

= 0.2 ×* 10% = 0.2 ×* 0.1 = 0.02 or 2%

Now, let's calculate the minimum percentage of women among all the employees surveyed by
summing up the contributions:

Minimum percentage of women among all employees = Percentage contribution from high
school diploma + Percentage contribution from Ph.D.

= 16% + 2%

108
= 18%

Therefore, the minimum percentage of women among all the employees surveyed is 18%.

113 - a
Total Profit/Loss = (Total Selling Price) - (Total Cost Price)

Total Selling Price = Rs 350 + Rs 290 + Rs 340 + Rs 360 + Rs 300 = Rs 1640

Total Cost Price = Rs 200 + Rs 180 + Rs 220 + Rs 250 + Rs 190 = Rs 1040

Total Profit = Rs 1640 - Rs 1040 = Rs 600

The total profit on the dress inventory of FashionEmporium is Rs 600.

114 - c

Total Original Revenue = Rs 480 + Rs 560 + Rs 400 + Rs 620 = Rs 2060

Total Markdown Revenue = Rs 300 + Rs 380 + Rs 240 + Rs 450 = Rs 1370

Total Decrease in Revenue = Total Original Revenue - Total Markdown Revenue Total Decrease
in Revenue = Rs 2060 - Rs 1370 = Rs 690

Total Percentage Decrease in Revenue = (Total Decrease in Revenue / Total Original Revenue
100 Total Percentage Decrease in Revenue = (690/2060) × 100 = 33.495% = 34%

Therefore, the total percentage decrease in revenue for ElectroTech after the markdown strategy
is approximately 33.50%.

115 - a

The total savings made by GroceryGalore on the bulk purchase of rice can be calculated as
follows:

Quantity of rice purchased = 500 kg

Original rate of rice = Rs 2.50 per kg

Discounted rate of rice = Rs 2.00 per kg

109
Total original cost = Rs 2.50 × 500 = Rs 1250

Total discounted cost = Rs 2.00 × 500 = Rs 1000

Total savings = Total original cost - Total discounted cost = Rs 1250 - Rs 1000 = Rs 250

So, GroceryGalore made a total savings of Rs 250 on the bulk purchase of rice.

116 - b

Total Loss = (Total Cost Price) - (Total Selling Price)

Total Cost Price = Rs 30 + Rs 40 + Rs 50 = Rs 120

Total Selling Price = Rs 20 + Rs 30 + Rs 40 = Rs 90

Total Loss = Rs 120 - Rs 90 = Rs 30

Therefore, the total loss incurred by TechHaven during the promotional event is Rs 30.

117 - c

If the average marks of the students in all four subjects are considered, then the overall average
mark of the class would be the average of the average marks in Mathematics, Science, English,
and History.

The average mark in Mathematics is 85, the average mark in Science is 75, the average mark in
English is 90, and the average mark in History is 80.

So, the overall average mark of the class would be:

(85 + 75 + 90 + 80) / 4 = 82.5

118 - c

If the class average mark in Mathematics increases by 5 points, then the new average mark in
Mathematics would be 85 + 5 = 90.

If the class average mark in Science decreases by 5 points, then the new average mark in Science
would be 75 - 5 = 70.

The average mark in English and History remains unchanged at 90 and 80, respectively.

110
So, the new overall average mark of the class in these four subjects would be:

(90 + 70 + 90 + 80) / 4 = 82.5

119 - a

To find the minimum average mark the class needs in another subject to maintain an average
mark of 80 across all subjects, we need to calculate the sum of the average marks in the current
subjects and subtract it from the desired total.

Current average marks = Mathematics (85) + Science (75) + English (90) + History (80) = 330

Desired total average marks = 80 x 5 (as there are 5 subjects) = 400

Minimum average mark needed in another subject = Desired total average marks - Current
average marks

= 400 - 330

= 70

Therefore, the class needs to achieve a minimum average mark of 70 in another subject to
maintain an average mark of 80 across all subjects. Option A is the correct answer.

120 - a

To find the new overall average mark of the class, we need to add 5 to each subject's average
mark and calculate the new average.

The sum of the new average marks is (85 + 5) + (75 + 5) + (90 + 5) + (80 + 5) = 90 + 80 + 95 +
85 = 350.

Since there are four subjects, the new overall average mark of the class is 350 divided by 4,
which equals 87.5.

111

You might also like